Ch. 24 Nursing Management of the Newborn at Risk: Acquired and Congenital Newborn Conditions, 23, OB COURSE POINT QUIZ 3/3, Ch20, Quiz 7, Chapter 19, Maternity Nursing, Chapter 18, Chapter 23, Chapter 24, *Chapter 21, Chapter 12, Chapter 11, Chapter...

¡Supera tus tareas y exámenes ahora con Quizwiz!

The nurse is preparing to initiate intravenous (IV) access on a patient in the active phase of labor. Which size IV cannula is best for this patient? a. 18-gauge b. 20-gauge c. 22-gauge d. 24-gauge

a. 18-gauge

During labor a vaginal examination should be performed only when necessary because of the risk of: a. infection. b. fetal injury. c. discomfort. d. perineal trauma.

a. infection.

A nurse is developing a plan of care for a newborn with omphalocele. Which of the following would the nurse include? A) Placing the newborn into a sterile drawstring bowel bag B) Using clean technique for dressing changes C) Preparing the newborn for incision and drainage D) Instituting gavage feedings

A

While providing care to a woman who is experiencing postpartum hemorrhage, the nurse weighs her perineal pads to estimate blood loss. The pad weighs 20 g. The nurse documents this as which amount? 5 mL 10 mL 15 mL 20 mL

20 mL explanation: When weighing perineal pads to determine blood loss, 1 gram of pad weight is equivalent to 1 mL of blood loss. A pad that weighs 20 g would indicate a 20-mL blood loss.

The first time the nurse sees a woman during pregnancy, her fundal height is palpable at the level of her umbilicus. This measurement is typical of what gestational age?

20 weeks The uterus expands to reach the height of the umbilicus by week 20. Before week 20 it is too low to be palpated, and after week 20 it may be beyond the umbilicus.

A baby who is declared AGA (appropriate for gestational age) falls in what weight percentile? 20th 9th 5th 95th

20th explanation: AGA infants are infants that fall between the 10th and 90th percentile for weight.

Twenty minutes after birth, a baby begins to move his head from side to side, making eye contact with the mother, and pushes his tongue out several times. The nurse interprets this as: A) A good time to initiate breast-feeding B) The period of decreased responsiveness preceding sleep C) The need to be alert for gagging and vomiting D) Evidence that the newborn is becoming chilled

A The newborn is demonstrating behaviors indicating the first period of reactivity, which usually begins at birth and lasts for the first 30 minutes. This is a good time to initiate breast-feeding. Decreased responsiveness occurs from 30 to 120 minutes of age and is characterized by muscle relaxation and diminished responsiveness to outside stimuli. There is no indication that the newborn may experience gagging or vomiting. Chilling would be evidenced by tachypnea, decreased activity, and hypotonia.

A woman with diabetes is considering becoming pregnant. She asks the nurse whether she will be able to take oral hypoglycemics when she is pregnant. The nurse's response is based on the understanding that oral hypoglycemics: A. show promising results, but more studies are needed to confirm their degree of safety. B. can be used as long as they control serum glucose levels. C. can be taken until the degeneration of the placenta occurs. D. are usually suggested primarily for women who develop gestational diabetes.

A. show promising results, but more studies are needed to confirm their degree of safety.

In a woman who is suspected of having a ruptured ectopic pregnancy, the nurse would expect to assess for which of the following as a priority? A) Hemorrhage B) Jaundice C) Edema D) Infection

Ans: A With a ruptured ectopic pregnancy, the woman is at high risk for hemorrhage. Jaundice, edema, and infection are not associated with a ruptured ectopic pregnancy.

Which assessment finding could indicate hemorrhage in the postpartum patient? a. Elevated pulse rate b. Elevated blood pressure c. Firm fundus at the midline d. Saturation of two perineal pads in 4 hours

a. Elevated pulse rate

At 5 minutes after birth, the nurse assesses that the neonate's heart rate is 96 bpm, respirations are spontaneous, with a strong cry, body posture is flexed with vigorous movement, reflexes are brisk, and there is cyanosis of the hands and feet. What Apgar score will the nurse assign? a. 7 b. 8 c. 9 d. 10

b. 8

The nurse assesses the amniotic fluid. Which characteristic presents the lowest risk of fetal complications? a. Bloody b. Clear with bits of vernix caseosa c Green and thick d. Yellow and cloudy with foul odor

b. Clear with bits of vernix caseosa

Which of the following behaviors would be applicable to a nursing diagnosis of risk for injury in a client who is in labor? a. Length of second-stage labor is 2 hours. b. Client has received an epidural for pain control during the labor process. c. Client is using breathing techniques during contractions to maximize pain relief. d. Client is receiving parenteral fluids during the course of labor to maintain hydration.

b. Client has received an epidural for pain control during the labor process.

The nurse assists the midwife during a vaginal examination of the client in labor. What does the nurse recognize as the primary reason that a vaginal exam is done at this time? a. To apply internal monitoring electrodes b. To assess for Goodell's sign c. To determine cervical dilation and effacement d. To determine strength of contractions

c. To determine cervical dilation and effacement

The nurse is preparing to perform Leopold's maneuvers. Why are Leopold's maneuvers used by practitioners? a. To determine the status of the membranes b. To determine cervical dilation and effacement c. To determine the best location to assess the fetal heart rate d. To determine whether the fetus is in the posterior position

c. To determine the best location to assess the fetal heart rate

A client at 40 weeks' gestation should be instructed to go to a hospital or birth center for evaluation when she experiences: a. fetal movement. b. irregular contractions for 1 hour. c. a trickle of fluid from the vagina. d. thick pink or dark red vaginal mucus.

c. a trickle of fluid from the vagina.

The nurse has given the newborn an Apgar score of 5. She should then: a. begin ventilation and compressions. b. do nothing except place the infant under a radiant warmer. c. observe the infant and recheck the score after 10 minutes. d. gently stimulate by rubbing the infant's back while administering O2.

d. gently stimulate by rubbing the infant's back while administering O2.

The nurse is teaching a group of parents who have preterm newborns about the differences between a full-term newborn and a preterm newborn. Which characteristic would the nurse describe as associated with a preterm newborn but not a term newborn? fewer visible blood vessels through the skin more subcutaneous fat in the neck and abdomen well-developed flexor muscles in the extremities greater surface area in proportion to weight

greater surface area in proportion to weight explanation: Preterm newborns have large body surface areas compared to weight, which allows an increased transfer of heat from their bodies to the environment. Preterm newborns often have thin transparent skin with numerous visible veins, minimal subcutaneous fat, and poor muscle tone.

The nurse prepares to administer a gavage feeding for a newborn with transient tachypnea based on the understanding that this type of feeding is necessary because: lactase enzymatic activity is not adequate. oxygen demands need to be reduced. renal solute lead must be considered. hyperbilirubinemia is likely to develop.

oxygen demands need to be reduced. explanation: For the newborn with transient tachypnea, the newborn's respiratory rate is high, increasing the oxygen demand. Thus, measures are initiated to reduce this demand. Gavage feedings are one way to do so. With transient tachypnea, enzyme activity and kidney function are not affected. This condition typically resolves within 72 hours. The risk for hyperbilirubinemia is not increased.

While reviewing a newborn's medical record, the nurse notes that the chest X-ray shows a ground glass pattern. The nurse interprets this as indicative of: respiratory distress syndrome. transient tachypnea of the newborn. asphyxia. persistent pulmonary hypertension.

respiratory distress syndrome. explanation: The chest X-ray of a newborn with RDS reveals a reticular (ground glass) pattern. For TTN, the chest X-ray shows lung overaeration and prominent perihilar interstitial markings and streakings. A chest X-ray for asphyxia would reveal possible structural abnormalities that might interfere with respiration, but the results are highly variable. An echocardiogram would be done to evaluate persistent pulmonary hypertension.

Review of a primiparous woman's labor and birth record reveals a prolonged second stage of labor and extended time in the stirrups. Based on an interpretation of these findings, the nurse would be especially alert for which condition? retained placental fragments hypertension thrombophlebitis uterine subinvolution

thrombophlebitis explanation: The woman is at risk for thrombophlebitis due to the prolonged second stage of labor, necessitating an increased amount of time in bed, and venous pooling that occurs when the woman's legs are in stirrups for a long period of time. These findings are unrelated to retained placental fragments, which would lead to uterine subinvolution, or hypertension.

After determining that a newborn is in need of resuscitation, which of the following would the nurse do first? A) Dry the newborn thoroughly B) Suction the airway C) Administer ventilations D) Give volume expanders

A

A 10-week pregnant woman with diabetes has a glycosylated hemoglobin (HbA1c) level of 13%. At this time the nurse should be most concerned about which of the following possible fetal outcomes? A) Congenital anomalies B) Incompetent cervix C) Placenta previa D) Abruptio placentae

A) Congenital anomalies A HbA1c level of 13% indicates poor glucose control. This, in conjunction with the woman being in the first trimester, increases the risk for congenital anomalies in the fetus. Elevated glucose levels are not associated with incompetent cervix, placenta previa, or abruptio placentae.

A nurse is explaining to the parents of a child with bladder exstrophy about the care their infant requires. Which of the following would the nurse include in the explanation? (Select all that apply.) A) Covering the area with a sterile, clear, nonadherent dressing B) Irrigating the surface with sterile saline twice a day C) Monitoring drainage through the suprapubic catheter D) Administering prescribed antibiotic therapy E) Preparing for surgical intervention in about 2 weeks

A, C, D

A nurse suspects that a preterm newborn is having problems with thermal regulation. Which of the following would support the nurse's suspicion? (Select all that apply.) A) Shallow, slow respirations B) Cyanotic hands and feet C) Irritability D) Hypertonicity E) Feeble cry

A,B,E

A newborn has been diagnosed with a Group B streptococcal infection shortly after birth. The nurse understands that the newborn most likely acquired this infection from which of the following? A) Improper handwashing B) Contaminated formula C) Nonsterile catheter insertion D) Mother's birth canal

D

If a pregnant woman's estimated date of delivery (EDD) is April 23, what was the first day of her last menstrual period (LMP), according to Nagele's rule?

July 16 According to Nagele's rule, the last menstrual period was July 16th. Take the LMP and add 7 days and subtract 3 months; if finding the LMP from the EDD, subtract 7 days and add 3 months.

The nurse notes a concerning fetal heart rate pattern for a patient in active labor. The health care provider has prescribed the placement of a Foley catheter. What priority nursing action will the nurse implement when placing the catheter? a. Place the catheter as quickly as possible. b. Place a small pillow under the patient's left hip. c. Omit the use of a cleansing agent, such as Betadine. d. Set up the catheter tray before positioning the patient.

b. Place a small pillow under the patient's left hip

The labor nurse is reviewing the cardinal maneuvers with a group of nursing students. Which maneuver will immediately follow the birth of the baby's head? a. Expulsion b. Restitution c. Internal rotation d. External rotation

b. Restitution

When performing newborn resuscitation, which action would the nurse do first? Intubate with an appropriate-sized endotracheal tube. Give chest compressions at a rate of 80 times per minute. Administer epinephrine intravenously. Suction the mouth and then the nose.

Suction the mouth and then the nose. explanation: After placing the newborn's head in a neutral position, the nurse would suction the mouth and then the nose. This is followed by ventilation, circulation (chest compressions), and administration of epinephrine

Assessment of a pregnant woman and her fetus reveals tachycardia and hypertension. There is also evidence suggesting vasoconstriction. The nurse would question the woman about use of which substance? A. marijuana B. heroin C. alcohol D. cocaine

D. cocaine

A nurse is assisting in the resuscitation of a newborn. The nurse would expect to stop resuscitation efforts when the newborn has no heartbeat and respiratory effort after which time frame? A) 5 minutes B) 10 minutes C) 15 minutes D) 20 minutes

B

The nurse prepares to assess a newborn who is considered to be large for gestational age (LGA). Which of the following would the nurse correlate with this gestational age variation? A) Strong, brisk motor skills B) Difficulty in arousing to a quiet alert state C) Birth weight of 7 lb 14 oz D) Wasted appearance of extremities

B

When describing the neurologic development of a newborn to his parents, the nurse would explain that it occurs in which fashion? A) Head-to-toe B) Lateral-to-medial C) Outward-to-inward D) Distal-caudal

A Neurologic development follows a cephalocaudal (head-to-toe) and proximal-distal (center to outside) pattern.

When teaching new parents about the sensory capabilities of their newborn, which sense would the nurse identify as being the least mature? A) Hearing B) Touch C) Taste D) Vision

D Vision is the least mature sense at birth. Hearing is well developed at birth, evidenced by the newborn's response to noise by turning. Touch is evidenced by the newborn's ability to respond to tactile stimuli and pain. A newborn can distinguish between sweet and sour by 72 hours of age.

The nurse administers vitamin K intramuscularly to the newborn based on which of the following rationales? A) Stop Rh sensitization B) Increase erythopoiesis C) Enhance bilirubin breakdown D) Promote blood clotting

D Vitamin K promotes blood clotting by increasing the synthesis of prothrombin by the liver. RhoGAM prevents Rh sensitization. Erythropoietin stimulates erythropoiesis. Phototherapy enhances bilirubin breakdown.

When assessing a pregnant woman with heart disease throughout the antepartal period, the nurse would be especially alert for signs and symptoms of cardiac decompensation at which time? A) 16 to 20 weeks' gestation B) 20 to 24 weeks' gestation C) 24 to 28 week's gestation D) 28 to 32 week's gestation

D) 28 to 32 week's gestation

A prenatal client in her second trimester is admitted to the maternity unit with painless, bright red vaginal bleeding. What test might the physician order? a)Alpha-fetoprotein (AFP) b)Contraction stress test (CST) c)Amniocentesis d)Ultrasound

D. An ultrasound for placenta location to rule out placenta previa would be ordered for a client who presents with painless, bright red vaginal bleeding. The ability to see the lower portion of the uterus and cervix with ultrasound is particularly important when vaginal bleeding is noted and placenta previa is the suspected cause. Alpha-fetoprotein (AFP) is a test used to screen for neural tube defects. A contraction stress test is ordered in the third trimester to evaluate the respiratory function of the placenta. Amniocentesis is a procedure used for genetic diagnosis or, in later pregnancy, for lung maturity studies.

When preparing a schedule of follow-up visits for a pregnant woman with chronic hypertension, which schedule would be most appropriate? A. bi-monthly visits until 36 weeks, then weekly visits B. monthly visits until 20 weeks, then bi-monthly visits C. monthly visits until 32 weeks, then bi-monthly visits D. bi-monthly visits until 28 weeks, then weekly visits

D. bi-monthly visits until 28 weeks, then weekly visits

A nurse is conducting a class for pregnant women with diabetes. Which factor would the nurse emphasize as being most important in helping to reduce the maternal/fetal/neonatal complications associated with pregnancy and diabetes? A. blood urea nitrogen level (BUN) within normal limits B. evaluation of retinopathy by an ophthalmologist C. stability of the woman's emotional and psychological status D. degree of glycemic control achieved during the pregnancy

D. degree of glycemic control achieved during the pregnancy

The nurse in the prenatal clinic assesses a 26-year-old client at 13 weeks gestation. Which presumptive (subjective) signs and symptoms of pregnancy should the nurse anticipate? a) Hegar's sign and quickening b) Ballottement and positive pregnancy test c) Chadwick's sign and uterine souffle d) Excessive fatigue and urinary frequency

D Excessive fatigue and urinary frequency both are presumptive (subjective) signs and symptoms of pregnancy. Hegar's sign, ballottement, a positive pregnancy test, Chadwick's sign, and uterine souffle are probable (objective) signs or symptoms of pregnancy.

A nurse is providing care to several pregnant women at the clinic. The nurse would screen for group B streptococcus infection in a client at: A) 16 weeks' gestation B) 28 week' gestation C) 32 weeks' gestation D) 36 weeks' gestation

D Feedback: According to the CDC guidelines, all pregnant women should be screened for group B streptococcus infection at 35 to 37 weeks' gestation.

The nurse is taking an initial history of a prenatal client. Which of the following, if detected by the nurse practitioner, would indicate a positive, or diagnostic sign of pregnancy? a) Positive pregnancy test b) Goodell's sign c) Uterine enlargement and amenorrhea d) Fetal heartbeat with at Doppler at 11 weeks gestation

D The positive signs of pregnancy are completely objective, cannot be confused with a pathologic state, and offer conclusive proof of pregnancy. The fetal heartbeat can be detected with an electronic Doppler device as early as weeks 10-12 of pregnancy. Pregnancy tests detect the presence of hCG in the maternal blood or urine. These are not considered a positive sign of pregnancy because other conditions can cause elevated hCG levels. Physical changes, like Godell's sign and uterine enlargement, can also have other causes and do not confirm pregnancy. The subjective changes of pregnancy, like amenorrhea, are the symptoms the woman experiences and reports. Because they can be caused by other conditions, they cannot be considered proof of pregnancy.

When performing newborn resuscitation, which action would the nurse do first? A) Intubate with an appropriate-sized endotracheal tube. B) Give chest compressions at a rate of 80 times per minute. C) Administer epinephrine intravenously. D) Suction the mouth and then the nose.

D After placing the newborn's head in a neutral position, the nurse would suction the mouth and then the nose. This is followed by ventilation, circulation (chest compressions), and administration of epinephrine.

The nurse places a newborn with jaundice under the phototherapy lights in the nursery to achieve which goal? A) Prevent cold stress B) Increase surfactant levels in the lungs C) Promote respiratory stability D) Decrease the serum bilirubin level

D Jaundice reflects elevated serum bilirubin levels; phototherapy helps to break down the bilirubin for excretion. Phototherapy has no effect on body temperature, surfactant levels, or respiratory stability.

The nurse institutes measure to maintain thermoregulation based on the understanding that newborns have limited ability to regulate body temperature because they: A) Have a smaller body surface compared to body mass B) Lose more body heat when they sweat than adults C) Have an abundant amount of subcutaneous fat all over D) Are unable to shiver effectively to increase heat production

D Newborns have difficulty maintaining their body heat through shivering and other mechanisms. They have a large body surface area relative to body weight and have limited sweating ability. Additionally, newborns lack subcutaneous fat to provide insulation.

The nurse is teaching a group of students about the differences between a full-term newborn and a preterm newborn. The nurse determines that the teaching is effective when the students state that the preterm newborn has: A) Fewer visible blood vessels through the skin B) More subcutaneous fat in the neck and abdomen C) Well-developed flexor muscles in the extremities D) Greater surface area in proportion to weight

D Preterm newborns have large body surface areas compared to weight, which allows an increased transfer of heat from their bodies to the environment. Preterm newborns often have thin transparent skin with numerous visible veins, minimal subcutaneous fat, and poor muscle tone.

Which of the following would alert the nurse to the possibility of respiratory distress in a newborn? A) Symmetrical chest movements B) Periodic breathing C) Respirations of 40 breaths/minute D) Sternal retractions

D Sternal retractions, cyanosis, tachypnea, expiratory grunting, and nasal flaring are signs of respiratory distress in a newborn. Symmetrical chest movements and a respiratory rate between 30 to 60 breaths/minute are typical newborn findings. Some newborns may demonstrate periodic breathing (cessation of breathing lasting 5 to 10 seconds without changes in color or heart rate) in the first few days of life.

A new mother reports that her newborn often spits up after feeding. Assessment reveals regurgitation. The nurse responds based on the understanding that this most likely is due to which of the following? A) Placing the newborn prone after feeding B) Limited ability of digestive enzymes C) Underdeveloped pyloric sphincter D) Relaxed cardiac sphincter

D The cardiac sphincter and nervous control of the stomach is immature, which may lead to uncoordinated peristaltic activity and frequent regurgitation. Placement of the newborn is unrelated to regurgitation. Most digestive enzymes are available at birth, but they are limited in their ability to digest complex carbohydrates and fats; this results in fatty stools, not regurgitation. Immaturity of the pharyngoesophageal sphincter and absence of lower esophageal peristaltic waves, not an underdeveloped pyloric sphincter, also contribute to the reflux of gastric contents.

Which of the following would alert the nurse to suspect that a preterm newborn is in pain? A) Bradycardia B) Oxygen saturation level of 94% C) Decreased muscle tone D) Sudden high-pitched cry

D The nurse should suspect pain if the newborn exhibits a sudden high-pitched cry, oxygen desaturation, tachycardia, and increased muscle tone.

While making rounds in the nursery, the nurse sees a 6-hour-old baby girl gagging and turning bluish. What would the nurse do first? A) Alert the physician stat and turn the newborn to her right side. B) Administer oxygen via facial mask by positive pressure. C) Lower the newborn's head to stimulate crying. D) Aspirate the oral and nasal pharynx with a bulb syringe.

D The nurse's first action would be to suction the oral and nasal pharynx with a bulb syringe to maintain airway patency. Turning the newborn to her right side will not alleviate the blockage due to secretions. Administering oxygen via positive pressure is not indicated at this time. Lowering the newborn's head would be inappropriate.

A nurse is assisting the anxious parents of a preterm newborn to cope with the situation. Which statement by the nurse would be least appropriate? A) "I'll be here to help you all along the way." B) "What has helped you to deal with stressful situations in the past?" C) "Let me tell you about what you will see when you visit your baby." D) "Forget about what's happened in the past and focus on the now."

D

The nurse is teaching a group of students about the differences between a full-term newborn and a preterm newborn. The nurse determines that the teaching is effective when the students state that the preterm newborn has: A) Fewer visible blood vessels through the skin B) More subcutaneous fat in the neck and abdomen C) Well-developed flexor muscles in the extremities D) Greater surface area in proportion to weight

D

When performing newborn resuscitation, which action would the nurse do first? A) Intubate with an appropriate-sized endotracheal tube. B) Give chest compressions at a rate of 80 times per minute. C) Administer epinephrine intravenously. D) Suction the mouth and then the nose.

D

Which of the following would alert the nurse to suspect that a preterm newborn is in pain? A) Bradycardia B) Oxygen saturation level of 94% C) Decreased muscle tone D) Sudden high-pitched cry

D

A pregnant client arrives for her second prenatal appointment. Her previous pregnancy ended at 19 weeks, and she has 3-year-old twins born at 30 weeks gestation. How will the nurse document her "G" and "L" for her records?

G3 L2 She has had two prior pregnancies and is pregnant now, total "G" = 3; she has twins from a prior pregnancy and one lost child for a total "L" = 2.

The nurse is reviewing the physical examination findings for a client who is to undergo labor induction. Which finding would indicate to the nurse that a woman's cervix is ripe in preparation for labor induction? posterior position firm closed shortened

Shortened explanation: A ripe cervix is shortened, centered (anterior), softened, and partially dilated. An unripe cervix is long, closed, posterior, and firm.

The nurse auscultates the fetal heart rate and determines a rate of 152 bpm. Which nursing intervention is appropriate? a. Inform the mother that the rate is normal. b. Reassess the fetal heart rate in 5 minutes because the rate is too high. c. Report the fetal heart rate to the physician or nurse-midwife immediately. d. Tell the mother that she is going to have a boy because the heart rate is fast.

a. Inform the mother that the rate is normal.

The nurse thoroughly dries the infant immediately after birth primarily to: a. reduce heat loss from evaporation. b. stimulate crying and lung expansion. c. increase blood supply to the hands and feet. d. remove maternal blood from the skin surface.

a. reduce heat loss from evaporation.

A woman who is gravida 3, para 2, enters the intrapartum unit. The most important nursing assessments are: a. contraction pattern, amount of discomfort, and pregnancy history. b. fetal heart rate, maternal vital signs, and the woman's nearness to birth. c. fast food intake, when labor began, and cultural practices the couple desires. d. identification of ruptured membranes, the woman's gravida and para, and her support person.

b. fetal heart rate, maternal vital signs, and the woman's nearness to birth.

The nurse frequently assesses the respiratory status of a preterm newborn based on the understanding that the newborn is at increased risk for respiratory distress syndrome because of: inability to clear fluids. immature respiratory control center. deficiency of surfactant. smaller respiratory passages.

deficiency of surfactant. explanation: A preterm newborn is at increased risk for respiratory distress syndrome (RDS) because of a surfactant deficiency. Surfactant helps to keep the alveoli open and maintain lung expansion. With a deficiency, the alveoli collapse, predisposing the newborn to RDS. An inability to clear fluids can lead to transient tachypnea. Immature respiratory control centers lead to an increased risk for apnea. Smaller respiratory passages led to an increased risk for obstruction.

The nurse is providing care to several pregnant women who may be scheduled for labor induction. The nurse identifies the woman with which Bishop score as having the best chance for a successful induction and vaginal birth? 11 8 6 3

11 explanation: The Bishop score helps identify women who would be most likely to achieve a successful induction. The duration of labor is inversely correlated with the Bishop score: a score over 8 indicates a successful vaginal birth. Therefore the woman with a Bishop score of 11 would have the greatest chance for success. Bishop scores of less than 6 usually indicate that a cervical ripening method should be used prior to induction.

The nurse assesses the uterine fundus and finds it to be halfway between the symphysis pubis and the umbilicus. The nurse would determine that this corresponds to how many gestational weeks?

16 At 16 weeks' gestation, the fundus is half-way between the symphysis and the umbilicus. At approximately 12 weeks, the fundus is at the top of the symphysis pubis and by 10 weeks the fundus should be at the level of the umbilicus.

Utilize the GTPAL system to classify a woman who is currently 18 weeks pregnant. This is her 4th pregnancy. She gave birth to one baby vaginally at 26 weeks who died, experienced a miscarriage, and has one living child who was delivered at 38 weeks gestation.

4, 1, 1, 1, 1 The GTPAL system is used to classifying pregnancy status. G = gravida, T= term, P = preterm, A = number of abortions, L= number of living children.

A newborn with severe meconium aspiration syndrome (MAS) is not responding to conventional treatment. Which of the following would the nurse anticipate as possibly necessary for this newborn? A) Extracorporeal membrane oxygenation (ECMO) B) Respiratory support with a ventilator C) Insertion of a laryngoscope for deep suctioning D) Replacement of an endotracheal tube via x-ray

A

After teaching the parents of a newborn with retinopathy of prematurity (ROP) about the disorder and treatment, which statement by the parents indicates that the teaching was successful? A) "Can we schedule follow-up eye examinations with the pediatric ophthalmologist now?" B) "We can fix the problem with surgery." C) "We'll make sure to administer eye drops each day for the next few weeks." D) "I'm sure the baby will grow out of it."

A

An LGA newborn has a blood glucose level of 30 mg/dL and is exhibiting symptoms of hypoglycemia. Which of the following would the nurse do next? A) Administer intravenous glucose immediately. B) Feed the newborn 2 ounces of formula. C) Initiate blow-by oxygen therapy. D) Place the newborn under a radiant warmer.

A

The nurse is assessing a preterm newborn who is in the neonatal intensive care unit (NICU. for signs and symptoms of overstimulation. Which of the following would the nurse be least likely to assess? A) Increased respirations B) Flaying hands C) Periods of apnea D) Decreased heart rate

A

When planning the care for an SGA newborn, which action would the nurse determine as a priority? A) Preventing hypoglycemia with early feedings B) Observing for respiratory distress syndrome C) Promoting bonding between the parents and the newborn D) Monitoring vital signs every 2 hours

A

Which action would be most appropriate for the nurse to take when a newborn has an unexpected anomaly at birth? A) Show the newborn to the parents as soon as possible while explaining the defect. B) Remove the newborn from the birthing area immediately. C) Inform the parents that there is nothing wrong at the moment. D) Tell the parents that the newborn must go to the nursery immediately.

A

While caring for a preterm newborn receiving oxygen therapy, the nurse monitors the oxygen therapy duration closely based on the understanding that the newborn is at risk for which of the following? A) Retinopathy of prematurity B) Metabolic acidosis C) Infection D) Cold stress

A

While reviewing a newborn's medical record, the nurse notes that the chest x-ray shows a ground glass pattern. The nurse interprets this as indicative of: A) Respiratory distress syndrome B) Transient tachypnea of the newborn C) Asphyxia D) Persistent pulmonary hypertension

A

The nurse is reviewing four prenatal charts. Which client would be an appropriate candidate for a contraction stress test (CST)? a)A client with intrauterine growth retardation b)A client with multiple gestation c)A client with an incompetent cervix d)A client with placenta previa

A A contraction stress test (CST) is indicated for a client with intrauterine growth retardation (IUGR), because it will assess the respiratory function of the placenta, which may be adversely affected by the conditions causing IUGR. The CST is contraindicated in third-trimester bleeding from placenta previa or marginal abruptio placentae, previous cesarean with classical incision (vertical incision in the fundus of the uterus), premature rupture of the membranes, incompetent cervix, anomalies of the maternal reproductive organs, history of preterm labor (if being done prior to term), or multiple gestation.

A pregnant woman with gestational diabetes comes to the clinic for a fasting blood glucose level. When reviewing the results, the nurse determines that which result indicates good glucose control? A) 90 mg/dL B) 100 mg/dL C) 110 mg /dL D) 120 mg/dL

A Feedback: For a pregnant woman with diabetes, the ADA (2012b) recommends maintaining a fasting blood glucose level below 95 mg/dL, with postprandial levels below 140 mg/dL and 2- hour postprandial levels below 120 mg/dL.

A nurse is preparing a teaching program for a group of pregnant women about preventing infections during pregnancy. When describing measures for preventing cytomegalovirus infection, which of the following would the nurse most likely include? A) Frequent handwashing B) Immunization C) Prenatal screening D) Antibody titer screening

A Feedback: Most women are asymptomatic and don't know they have been exposed to CMV. Prenatal screening for CMV infection is not routinely performed. Since there is no therapy that prevents or treats CMV infections, nurses are responsible for educating and supporting childbearing-age women at risk for CMV infection. Stressing the importance of good handwashing and use of sound hygiene practices can help to reduce transmission of the virus. There is no immunization for CMV. Antibody titer levels would be useful for identifying women at risk for rubella.

Because a pregnant client's diabetes has been poorly controlled throughout her pregnancy, the nurse would be alert for which of the following in the neonate at birth? A) Macrosomia B) Hyperglycemia C) Low birth weight D) Hypobilirubinemia

A Feedback: Poorly controlled diabetes during pregnancy can result in macrosomia due to hyperinsulinemia stimulated by fetal hyperglycemia. Typically the neonate is hypoglycemic due to the ongoing hyperinsulinemia that occurs after the placenta is removed. Infants of diabetic women typically are large and are at risk for hyperbilirubinemia due to excessive red blood cell breakdown.

Understanding the transition from intrauterine to extrauterine life, what intervention is most appropriate when working with an infant of a diabetic mother? a)Frequent blood glucose checks b)Obtain lab work to look for infection. c)Administer IV fluids. d)Place under radiant warmer bed immediately.

A Lab work, IV fluids, and the radiant warmer bed may all be required for interventions for the infant of a diabetic mother, if the infant is experiencing signs of respiratory distress or sepsis. Frequent blood glucose checks need to be completed to ensure that blood glucose levels are being maintained.

As relates to the father's acceptance of the pregnancy and preparation for childbirth, the maternity nurse should know that: A. The father goes through three phases of acceptance of his own. B. The father's attachment to the fetus cannot be as strong as that of the mother because it does not start until after birth. C. In the last 2 months of pregnancy, most expectant fathers suddenly get very protective of their established lifestyle and resist making changes to the home. D. Typically men remain ambivalent about fatherhood right up to the birth of their child. A

A (A father typically goes through three phases of development to reach acceptance of fatherhood: the announcement phase, the moratorium phase, and the focusing phase. The father-child attachment can be as strong as the mother-child relationship and can also begin during pregnancy. In the last 2 months of pregnancy, many expectant fathers work hard to improve the environment of the home for the child. Typically the expectant father's ambivalence ends by the first trimester, and he progresses to adjusting to the reality of the situation and then to focusing on his role.)

When counseling a mother about the immunologic properties of breast milk, the nurse would emphasize breast milk as a major source of which immunoglobulin? A) IgA B) IgG C) IgM D) IgE

A A major source of IgA is human breast milk. IgG, found in serum and interstitial fluid, crosses the placenta beginning at approximately 20 to 22 weeks' gestation. IgM is found in blood and lymph fluid and levels are generally low at birth unless there is a congenital intrauterine infection. IgE is not found in breast milk and does not play a major role in defense in the newborn.

After the birth of a newborn, which of the following would the nurse do first to assist in thermoregulation? A) Dry the newborn thoroughly. B) Put a hat on the newborn's head. C) Check the newborn's temperature. D) Wrap the newborn in a blanket.

A Drying the newborn immediately after birth using warmed blankets is essential to prevent heat loss through evaporation. Then the nurse would place a cap on the baby's head and wrap the newborn. Assessing the newborn's temperature would occur once these measures were initiated to prevent heat loss.

A client expresses concern that her 2-hour-old newborn is sleepy and difficult to awaken. The nurse explains that this behavior indicates which of the following? A) Normal progression of behavior B) Probable hypoglycemia C) Physiological abnormality D) Inadequate oxygenation

A From 30 to 120 minutes of age, the newborn enters the second stage of transition, that of sleep or a decrease in activity. More information would be needed to determine if hypoglycemia, a physiologic abnormality, or inadequate oxygenation was present.

An LGA newborn has a blood glucose level of 23 mg/dL. Which of the following would the nurse do next? A) Administer intravenous glucose immediately. B) Feed the newborn 2 ounces of formula. C) Initiate blow-by oxygen therapy. D) Place the newborn under a radiant warmer.

A If an LGA newborn's blood glucose level is below 25 mg/dL, the nurse should institute immediate treatment with intravenous glucose regardless of the clinical symptoms. Oral feedings would be used to maintain the newborn's glucose level above 40 mg/dL. Blow-by oxygen would have no effect on glucose levels; it may be helpful in promoting oxygenation. Placing the newborn under a radiant warmer would be a more appropriate measure for cold stress.

While caring for a preterm newborn receiving oxygen therapy, the nurse monitors the oxygen level and duration closely based on the understanding that the newborn is at risk for which of the following? A) Retinopathy of prematurity B) Metabolic acidosis C) Infection D) Cold stress

A Oxygen therapy has been implicated in the pathogenesis of retinopathy of prematurity (ROP). Therefore, the nurse monitors the newborn's oxygen therapy closely. Metabolic acidosis may occur due to anaerobic metabolism used for heat production. Infection may occur for numerous reasons, but they are unrelated to oxygen therapy. Cold stress results from problems due to the preterm newborn's inadequate supply of brown fat, decreased muscle tone, and large body surface area.

Assessment of a newborn reveals a heart rate of 180 beats/minute. To determine whether this finding is a common variation rather than a sign of distress, what else does the nurse need to know? A) How many hours old is this newborn? B) How long ago did this newborn eat? C) What was the newborn's birthweight? D) Is acrocyanosis present?

A The typical heart rate of a newborn ranges from 120 to 160 beats per minute with wide fluctuation during activity and sleep. Typically heart rate is assessed every 30 minutes until stable for 2 hours after birth. The time of the newborn's last feeding and his birthweight would have no effect on his heart rate. Acrocyanosis is a common normal finding in newborns.

When planning the care for a SGA newborn, which action would the nurse determine as a priority? A) Preventing hypoglycemia with early feedings B) Observing for respiratory distress syndrome C) Promoting bonding between the parents and the newborn D) Monitoring vital signs every 2 hours

A With the loss of the placenta at birth, the newborn must now assume control of glucose homeostasis. This is achieved by early oral intermittent feedings. Observing for respiratory distress, promoting bonding, and monitoring vital signs, although important, are not the priority for this newborn.

A nurse is preparing a teaching program for a group of pregnant women about preventing infections during pregnancy. When describing measures for preventing cytomegalovirus infection, which of the following would the nurse most likely include? A) Frequent handwashing B) Immunization C) Prenatal screening D) Antibody titer screening

A) Frequent handwashing Most women are asymptomatic and don't know they have been exposed to CMV. Prenatal screening for CMV infection is not routinely performed. Since there is no therapy that prevents or treats CMV infections, nurses are responsible for educating and supporting childbearing-age women at risk for CMV infection. Stressing the importance of good handwashing and use of sound hygiene practices can help to reduce transmission of the virus. There is no immunization for CMV. Antibody titer levels would be useful for identifying women at risk for rubella.

Because a pregnant client's diabetes has been poorly controlled throughout her pregnancy, the nurse would be alert for which of the following in the neonate at birth? A) Macrosomia B) Hyperglycemia C) Low birthweight D) Hypobilirubinemia

A) Macrosomia Poorly controlled diabetes during pregnancy can result in macrosomia due to hyperinsulinemia stimulated by fetal hyperglycemia. Typically the neonate is hypoglycemic due to the ongoing hyperinsulinemia that occurs after the placenta is removed. Infants of diabetic women typically are large and are at risk for hyperbilirubinemia due to excessive red blood cell breakdown.

A nurse is teaching a group of student nurses about amniotic fluid. Which of the following statements by the student nurse reflects an understanding of the fetus's contribution to the quality of amniotic fluid? Select all that apply. a) "The fetus contributes to the volume of amniotic fluid by excreting urine." b) "Approximately 400 mL of amniotic fluid flows out of the fetal lungs each day." c) "The fetus swallows about 600 mL of the fluid in 24 hours." d) "A fetus can move freely and develop normally, even if there is no amniotic fluid."

A, B, C "The fetus contributes to the volume of amniotic fluid by excreting urine." Approximately 400 mL of amniotic fluid flows out of the fetal lungs each day. The fetus swallows about 600 mL of the fluid in 24 hours. A normal volume of amniotic fluid is necessary for good fetal movement. Normal movement is necessary for good musculoskeletal development.

After teaching a pregnant woman with iron deficiency anemia about nutrition, the nurse determines that the teaching was successful when the woman identifies which of the following as being good sources of iron in her diet? (Select all that apply.) A) Dried fruits B) Peanut butter C) Meats D) Milk E) White bread

A, B, C Feedback: Foods high in iron include meats, green leafy vegetables, legumes, dried fruits, whole grains, peanut butter, bean dip, whole-wheat fortified breads and cereals.

A pregnant client asks why ultrasound is used so frequently during pregnancy. The nurse's response is based on her knowledge that the advantages of ultrasound include which of the following? Select all that apply. a)"It is noninvasive and painless." b)"It can be used to estimate gestational age." c)"Results are immediate." d)"The ultrasound is the only test to determine gender."

A, B, C The ability to establish fetal age accurately by ultrasound is lost in the third trimester because fetal growth is not as uniform as it is in the first two trimesters; however, ultrasound can be used to approximate gestational age within 1-3 weeks' accuracy during the third trimester. A comprehensive ultrasound is used to detect anatomical defects, not gestational age. Ultrasound is not used to determine gender.

The nurse is teaching a prenatal client about chorionic villus sampling (CVS). The nurse correctly teaches the client that risks related to CVS include which of the following? Select all that apply. a)Intrauterine infection b)Rupture of membranes c)Maternal hypertension d)Spontaneous abortion

A, B, D Risks of CVS include failure to obtain tissue, rupture of membranes, leakage of amniotic fluid, bleeding, intrauterine infection, maternal tissue contamination of the specimen, and Rh alloimmunization. CVS testing has a higher rate of spontaneous abortion than amniocentesis. Other complications include fetal limb defects and abnormalities of the fetal face and jaw.

A woman has just moved to the United States from Mexico. She is 3 months pregnant and has arrived for her first prenatal visit. During her assessment interview, you discover that she has not had any immunizations. Which immunizations should she receive at this point in her pregnancy (Select all that apply)? A. Tetanus B. Diphtheria C. Chickenpox D. Rubella E. Hepatitis B

A, B, E (Immunization with live or attenuated live viruses is contraindicated during pregnancy because of potential teratogenicity. Vaccines consisting of killed viruses may be used. Immunizations that may be administered during pregnancy include tetanus, diphtheria, recombinant hepatitis B, and rabies vaccines. Live-virus vaccines include those for measles (rubeola and rubella), chickenpox, and mumps.)

A nurse is teaching a group of first-trimester prenatal clients about the discomforts of pregnancy. A client asks the nurse, "What causes my nausea and vomiting?" The nurse responds indicating which of the following as being contributing factors to first-trimester emesis? Select all that apply. a) Human chorionic gonadotropin b) Estrogen c) Alterations in carbohydrate metabolism d) Prostaglandins

A, C Nausea and vomiting are common during the first trimester because of elevated human chorionic gonadotropin levels and changed carbohydrate metabolism. Estrogen stimulates the growth of the uterus and breast tissue. Prostaglandins stimulate uterine contractions.

The nurse is assessing a newborn and suspects that the newborn was exposed to drugs in utero because the newborn is exhibiting signs of neonatal abstinence syndrome. Which of the following would the nurse expect to assess? (Select all that apply.) A) Tremors B) Diminished sucking C) Regurgitation D) Shrill, high-pitched cry E) Hypothermia F) Frequent sneezing

A, C, D, F

A newborn was diagnosed with a congenital heart defect and will undergo surgery at a later time. The nurse is teaching the parents about signs and symptoms that need to be reported. The nurse determines that the parents have understood the instructions when they state that they will report which of the following? (Select all that apply.) A) Weight loss B) Pale skin C) Fever D) Absence of edema E) Increased respiratory rate

A, C, E

A prenatal client at 22 weeks gestation is scheduled for an amniocentesis. Which nursing action would apply to any client undergoing this procedure? Select all that apply. a)Assess for bleeding. b)Administer Rh immune globulin to the client. c)Cleanse skin with alcohol. d)Assess vital signs and fetal heart rate.

A, D The skin is cleaned with a betadine solution. The use of a local anesthesia at the needle insertion site is optional. A 22-gauge needle is then inserted into the uterine cavity and amniotic fluid is withdrawn. After 15-20 mL of fluid has been removed, the needle is withdrawn and the site is assessed for streaming (movement of fluid), which is an indication of bleeding. The fetal heart rate and maternal vital signs are then assessed. Rh immune globulin is given only to all Rh-negative women.

A nursing instructor is describing common problems associated with preterm birth. When describing the preterm newborn's risk for perinatal asphyxia, the instructor includes which of the following as contributing to the newborn's risk? (Select all that apply.) A) Surfactant deficiency B) Placental deprivation C) Immaturity of the respiratory control centers D) Decreased amounts of brown fat E) Depleted glycogen stores

A,C

A nurse is assessing a newborn who has been classified as small for gestational age. Which of the following would the nurse expect to find? (Select all that apply.) A) Wasted extremity appearance B) Increased amount of breast tissue C) Sunken abdomen D) Adequate muscle tone over buttocks E) Narrow skull sutures

A,C,E

What factors influence the outcomes of the at-risk newborn? Select all that apply. a)Birth weight b)Gestational age c)Type and length of newborn illness d)Environmental factors e)Maternal factors

All are correct. Maternal factors such as age and parity, newborn weight, and gestational age also influence outcomes, as do environmental factors such as exposure to environmental dangers (toxic chemicals and illicit drugs). Evaluation; Physiological Integrity; Analysis

A pregnant woman is receiving misoprostol to ripen her cervix and induce labor. The nurse assesses the woman closely for which of the following? A) Uterine hyperstimulation B) Headache C) Blurred vision D) Hypotension

Ans: A A major adverse effect of the obstetric use of Cytotec is hyperstimulation of the uterus, which may progress to uterine tetany with marked impairment of uteroplacental blood flow, uterine rupture (requiring surgical repair, hysterectomy, and/or salpingo-oophorectomy), or amniotic fluid embolism. Headache, blurred vision, and hypotension are associated with magnesium sulfate.

After teaching a woman who has had an evacuation for a hydatidiform mole (molar pregnancy) about her condition, which of the following statements indicates that the nurse's teaching was successful? A) "I will be sure to avoid getting pregnant for at least 1 year." B) "My intake of iron will have to be closely monitored for 6 months." C) "My blood pressure will continue to be increased for about 6 more months." D) "I won't use my birth control pills for at least a year or two."

Ans: A After evacuation of a hydatidiform mole, long-term follow-up is necessary to make sure any remaining trophoblastic tissue does not become malignant. Serial hCG levels are monitored closely for 1 year and the client is urged to avoid pregnancy for 1 year because it can interfere with the monitoring of hCG levels. Iron intake and blood pressure are not important aspects of follow-up after evacuation of a hydatidiform mole. Use of a reliable contraceptive is strongly recommended so that pregnancy is avoided.

A woman who delivered a healthy newborn several hours ago asks the nurse, "Why am I perspiring so much?" The nurse integrates knowledge that a decrease in which hormone plays a role in this occurrence? A) Estrogen B) hCG C) hPL D) Progesterone

Ans: A Although hCG, hPL, and progesterone decline rapidly after birth, decreased estrogen levels are associated with breast engorgement and with the diuresis of excess extracellular fluid accumulated during pregnancy.

A woman is to undergo an amnioinfusion. Which statement would be most appropriate to include when teaching the woman about this procedure? A) "You'll need to stay in bed while you're having this procedure." B) "We'll give you an analgesic to help reduce the pain." C) "After the infusion, you'll be scheduled for a cesarean birth." D) "A suction cup is placed on your baby's head to help bring it out."

Ans: A An amnioinfusion involves the instillation of a volume of warmed, sterile normal saline or Ringer's lactate into the uterus via an intrauterine pressure catheter. The client must remain in bed during the procedure. The use of analgesia is unrelated to this procedure. A cesarean birth is necessary only if the FHR does not improve after the amnioinfusion. Application of a suction cup to the head of the fetus refers to a vacuum-assisted birth.

The nurse is making a follow-up home visit to a woman who is 12 days postpartum. Which of the following would the nurse expect to find when assessing the client's fundus? A) Cannot be palpated B) 2 cm below the umbilicus C) 6 cm below the umbilicus D) 10 cm below the umbilicus

Ans: A By the end of 10 days, the fundus usually cannot be palpated because it has descended into the true pelvis.

When assessing a woman in her first trimester, which emotional response would the nurse most likely expect to find? A) Ambivalence B) Introversion C) Acceptance D) Emotional lability

Ans: A During the first trimester, the pregnant woman commonly experiences ambivalence, with conflicting feelings at the same time. Introversion heightens during the first and third trimesters when the woman's focus is on behaviors that will ensure a safe and healthy pregnancy outcome. Acceptance usually occurs during the second trimester. Emotional lability (mood swings) is characteristic throughout a woman's pregnancy.

The nurse interprets which of the following as evidence that a client is in the taking-in phase? A) Client states, "He has my eyes and nose." B) Client shows interest in caring for the newborn. C) Client performs self-care independently. D) Client confidently cares for the newborn.

Ans: A During the taking-in phase, new mothers when interacting with their newborns spend time claiming the newborn and touching him or her, commonly identifying specific features in the newborn such as "he has my nose" or "his fingers are long like his father's." Independence in self-care and interest in caring for the newborn are typical of the taking-hold phase. Confidence in caring for the newborn is demonstrated during the letting-go phase.

A postpartum client is experiencing subinvolution. When reviewing the woman's labor and birth history, which of the following would the nurse identify as being least significant to this condition? A) Early ambulation B) Prolonged labor C) Large fetus D) Use of anesthetics

Ans: A Factors that inhibit involution include prolonged labor and difficult birth, incomplete expulsion of amniotic membranes and placenta, uterine infection, overdistention of uterine muscles (such as by multiple gestation, hydramnios, or large singleton fetus), full bladder (which displaces the uterus and interferes with contractions), anesthesia (which relaxes uterine muscles), and close childbirth spacing. Factors that facilitate uterine involution include complete expulsion of amniotic membranes and placenta at birth, complication-free labor and birth process, breast-feeding, and early ambulation.

A woman who is 42 weeks pregnant comes to the clinic. Which of the following would be most important? A) Determining an accurate gestational age B) Asking her about the occurrence of contractions C) Checking for spontaneous rupture of membranes D) Measuring the height of the fundus

Ans: A Incorrect dates account for the majority of prolonged or postterm pregnancies; many women have irregular menses and thus cannot identify the date of their last menstrual period accurately. Therefore, accurate gestational dating via ultrasound is essential. Asking about contractions and checking for ruptured membranes, although important assessments, would be done once the gestational age is confirmed. Measuring the height of the fundus would be unreliable because after 36 weeks, the fundal height drops due to lightening and may no longer correlate with gestational weeks.

Assessment of a pregnant woman reveals a pigmented line down the middle of her abdomen. The nurse documents this as which of the following? A) Linea nigra B) Striae gravidarum C) Melasma D) Vascular spiders

Ans: A Linea nigra refers to the darkened line of pigmentation down the middle of the abdomen in pregnant women. Striae gravidarum refers to stretch marks, irregular reddish streaks on the abdomen, breasts, and buttocks. Melasma refers to the increased pigmentation on the face, also known as the mask of pregnancy. Vascular spiders are small, spiderlike blood vessels that appear usually above the waist and on the neck, thorax, face, and arms.

The nurse strokes the lateral sole of the newborn's foot from the heel to the ball of the foot when evaluating which reflex? A) Babinski B) Tonic neck C) Stepping D) Plantar grasp

Ans: A The Babinski reflex is elicited by stroking the lateral sole of the newborn's foot from the heel toward and across the ball of the foot. The tonic neck reflex is tested by having the newborn lie on his back and then turn his head to one side. The stepping reflex is elicited by holding the newborn upright and inclined forward with the soles of the feet on a flat surface. The plantar grasp reflex is elicited by placing a finger against the area just below the newborn's toes.

The nurse is providing care to several pregnant women who may be scheduled for labor induction. The nurse identifies the woman with which Bishop score as having the best chance for a successful induction and vaginal birth? A) 11 B) 8 C) 6 D) 3

Ans: A The Bishop score helps identify women who would be most likely to achieve a successful induction. The duration of labor is inversely correlated with the Bishop score: a score over 8 indicates a successful vaginal birth. Therefore the woman with a Bishop score of 11 would have the greatest chance for success. Bishop scores of less than 6 usually indicate that a cervical ripening method should be used prior to induction.

Which of the following would the nurse have readily available for a client who is receiving magnesium sulfate to treat severe preeclampsia? A) Calcium gluconate B) Potassium chloride C) Ferrous sulfate D) Calcium carbonate

Ans: A The antidote for magnesium sulfate is calcium gluconate, and this should be readily available in case the woman has signs and symptoms of magnesium toxicity.

In a client's seventh month of pregnancy, she reports feeling ìdizzy, like I'm going to pass out, when I lie down flat on my back.î The nurse integrates which of the following in to the explanation? A) Pressure of the gravid uterus on the vena cava B) A 50% increase in blood volume C) Physiologic anemia due to hemoglobin decrease D) Pressure of the presenting fetal part on the diaphragm

Ans: A The client is describing symptoms of supine hypotension syndrome, which occurs when the heavy gravid uterus falls back against the superior vena cava in the supine position. The vena cava is compressed, reducing venous return, cardiac output, and blood pressure, with increased orthostasis. The increased blood volume and physiologic anemia are unrelated to the client's symptoms. Pressure on the diaphragm would lead to dyspnea.

When assessing a newborn 1 hour after birth, the nurse measures an axillary temperature of 95.8° F, an apical pulse of 114 beats/minute, and a respiratory rate of 60 breaths/minute. Which nursing diagnosis takes highest priority? A) Hypothermia related to heat loss during birthing process B) Impaired parenting related to addition of new family member C) Risk for deficient fluid volume related to insensible fluid loss D) Risk for infection related to transition to extrauterine environment

Ans: A The newborn's heart rate is slightly below the accepted range of 120 to 160 beats/minute; the respiratory rate is at the high end of the accepted range of 30 to 60 breaths per minute. However, the newborn's temperature is significantly below the accepted range of 97.7 to 99.5° F. Therefore, the priority nursing diagnosis is hypothermia. There is no information to suggest impaired parenting. Additional information is needed to determine if there is a risk for deficient fluid volume or a risk for infection.

While changing a female newborn's diaper, the nurse observes a mucus-like, slightly bloody vaginal discharge. Which of the following would the nurse do next? A) Document this as pseudomenstruation B) Notify the practitioner immediately C) Obtain a culture of the discharge D) Inspect for engorgement

Ans: A The nurse should assess pseudomenstruation, a vaginal discharge composed of mucus mixed with blood, which may be present during the first few weeks of life. This discharge requires no treatment. The discharge is a normal finding and thus does not need to be reported immediately. It is not an indication of infection. The female genitalia normally will be engorged, so assessing for engorgement is not indicated.

The nurse is inspecting the external genitalia of a male newborn. Which of the following would alert the nurse to a possible problem? A) Limited rugae B) Large scrotum C) Palpable testes in scrotal sac D) Absence of engorgement

Ans: A The scrotum usually appears relatively large and should be pink in white neonates and dark brown in neonates of color. Rugae should be well formed and should cover the scrotal sac. There should not be bulging, edema, or discoloration. Testes should be palpable in the scrotal sac and feel firm and smooth and be of equal size on both sides of the scrotal sac.

Assessment of a newborn reveals a heart rate of 180 beats/minute. To determine whether this finding is a common variation rather than a sign of distress, what else does the nurse need to know? A) How many hours old is this newborn? B) How long ago did this newborn eat? C) What was the newborn's birth weight? D) Is acrocyanosis present?

Ans: A The typical heart rate of a newborn ranges from 120 to 160 beats per minute with wide fluctuation during activity and sleep. Typically heart rate is assessed every 30 minutes until stable for 2 hours after birth. The time of the newborn's last feeding and his birth weight would have no effect on his heart rate. Acrocyanosis is a common normal finding in newborns.

Prior to discharging a 24-hour-old newborn, the nurse assesses her respiratory status. Which of the following would the nurse expect to assess? A) Respiratory rate 45, irregular B) Costal breathing pattern C) Nasal flaring, rate 65 D) Crackles on auscultation

Ans: A Typically, respirations in a 24-hour-old newborn are symmetric, slightly irregular, shallow, and unlabored at a rate of 30 to 60 breaths/minute. The breathing pattern is primarily diaphragmatic. Nasal flaring, rates above 60 breaths per minute, and crackles suggest a problem.

The nurse is assessing a preterm newborn who is in the neonatal intensive care unit (NICU) for signs and symptoms of overstimulation. Which of the following would the nurse be least likely to assess? A) Increased respirations B) Flaying hands C) Periods of apnea D) Decreased heart rate

Ans: A Feedback: Conversely, overstimulation may have negative effects by reducing oxygenation and causing stress. A newborn reacts to stress by flaying the hands or bringing an arm up to cover the face. When overstimulated, such as by noise, lights, excessive handling, alarms, and procedures, and stressed, heart and respiratory rates decrease and periods of apnea or bradycardia may occur.

A nurse is developing a plan of care for a preterm infant experiencing respiratory distress. Which of the following would the nurse be least likely to include in this plan? A) Stimulate the infant with frequent handling. B) Keep the newborn in a warmed isolette. C) Administer oxygen using an oxygen hood. D) Give gavage or continuous tube feedings.

Ans: A Feedback: For the preterm infant experiencing respiratory distress, the nurse would expect to handle the newborn as little as possible to reduce oxygen requirements. Other appropriate interventions include keeping the infant warm, preferably in a warmed isolette to conserve the baby's energy and prevent cold stress; administer oxygen using an oxygen hood; and provide energy through calories via intravenous dextrose or gavage or continuous tube feedings to prevent hypoglycemia.

After determining that a newborn is in need of resuscitation, which of the following would the nurse do first?4A) Dry the newborn thoroughly B) Suction the airway C) Administer ventilations D) Give volume expanders

Ans: A Feedback: If resuscitation is need, the nurse must first stabilize the newborn by drying the newborn thoroughly with a warm towel and provide warmth by placing him or her under a radiant heater to prevent rapid heat loss. Next the newborn's head is placed in a neutral position to open the airway and the airway is cleared with a bulb syringe or suction catheter. Breathing is stimulated. Often handling and rubbing the newborn with a dry towel may be all that is needed to stimulate respirations. Next ventilations and then chest compressions are done. Administration of epinephrine and/or volume expanders is the last step.

While caring for a preterm newborn receiving oxygen therapy, the nurse monitors the oxygen therapy duration closely based on the understanding that the newborn is at risk for which of the following? A) Retinopathy of prematurity B) Metabolic acidosis C) Infection D) Cold stress

Ans: A Feedback: Oxygen therapy has been linked the pathogenesis of retinopathy of prematurity and is associated with the duration of oxygen use rather than the concentration of oxygen. Therefore, the nurse monitors the newborn's oxygen therapy closely. Metabolic acidosis may occur due to anaerobic metabolism used for heat production. Infection may occur for numerous reasons, but they are unrelated to oxygen therapy. Cold stress results from problems due to the preterm newborn's inadequate supply of brown fat, decreased muscle tone, and large body surface area.

After teaching the parents of a newborn with retinopathy of prematurity (ROP) about the disorder and treatment, which statement by the parents indicates that the teaching was successful? A) "Can we schedule follow-up eye examinations with the pediatric ophthalmologist now?" B) "We can fix the problem with surgery." C) "We'll make sure to administer eye drops each day for the next few weeks." D) "I'm sure the baby will grow out of it."

Ans: A Feedback: Parents of a newborn with suspected retinopathy of prematurity (ROP) should schedule follow-up vision screenings with a pediatric ophthalmologist every 2 to 3 weeks, depending on the severity of the findings at the initial examination.

While reviewing a newborn's medical record, the nurse notes that the chest x-ray shows a ground glass pattern. The nurse interprets this as indicative of: A) Respiratory distress syndrome B) Transient tachypnea of the newborn C) Asphyxia D) Persistent pulmonary hypertension

Ans: A Feedback: The chest x-ray of a newborn with RDS reveals a reticular (ground glass) pattern. For TTN, the chest x-ray shows lung overaeration and prominent perihilar interstitial markings and streakings. A chest x-ray for asphyxia would reveal possible structural abnormalities that might interfere with respiration, but the results are highly variable. An echocardiogram would be done to evaluate persistent pulmonary hypertension.

Which action would be most appropriate for the nurse to take when a newborn has an unexpected anomaly at birth? A) Show the newborn to the parents as soon as possible while explaining the defect. B) Remove the newborn from the birthing area immediately. C) Inform the parents that there is nothing wrong at the moment. D) Tell the parents that the newborn must go to the nursery immediately.

Ans: A Feedback: When an anomaly is identified at or after birth, parents need to be informed promptly and given a realistic appraisal of the severity of the condition, the prognosis, and treatment options so that they can participate in all decisions concerning their child. Removing the newborn from the area or telling them that the newborn needs to go to the nursery immediately is inappropriate and would only add to the parents' anxieties and fears. Telling them that nothing is wrong is inappropriate because it violates their right to know.

When planning the care for an SGA newborn, which action would the nurse determine as a priority? A) Preventing hypoglycemia with early feedings B) Observing for respiratory distress syndrome C) Promoting bonding between the parents and the newborn D) Monitoring vital signs every 2 hours

Ans: A Feedback: With the loss of the placenta at birth, the newborn must now assume control of glucose homeostasis. This is achieved by early oral intermittent feedings. Observing for respiratory distress, promoting bonding, and monitoring vital signs, although important, are not the priority for this newborn.

A group of students are reviewing the signs of pregnancy. The students demonstrate understanding of the information when they identify which as presumptive signs? (Select all that apply. A) Amenorrhea B) Nausea C) Abdominal enlargement D) Braxton-Hicks contractions E) Fetal heart sounds

Ans: A, B Presumptive signs include amenorrhea, nausea, breast tenderness, urinary frequency and fatigue. Abdominal enlargement and Braxton-Hicks contractions are probable signs of pregnancy. Fetal heart sounds are a positive sign of pregnancy.

A group of nursing students are reviewing information about cesarean birth. The students demonstrate understanding of the information when they identify which of the following as an appropriate indication? (Select all that apply.) A) Active genital herpes infection B) Placenta previa C) Previous cesarean birth D) Prolonged labor E) Fetal distress

Ans: A, B, C, E The leading indications for cesarean birth are previous cesarean birth, breech presentation, dystocia, and fetal distress. Examples of specific indications include active genital herpes, fetal macrosomia, fetopelvic disproportion, prolapsed umbilical cord, placental abnormality (placenta previa or abruptio placentae), previous classic uterine incision or scar, gestational hypertension, diabetes, positive HIV status, and dystocia. Fetal indications include malpresentation (nonvertex presentation), congenital anomalies (fetal neural tube defects, hydrocephalus, abdominal wall defects), and fetal distress.

While assessing a pregnant woman, the nurse suspects that the client may be at risk for hydramnios based on which of the following? (Select all that apply.) A) History of diabetes B) Complaints of shortness of breath C) Identifiable fetal parts on abdominal palpation D) Difficulty obtaining fetal heart rate E) Fundal height below that for expected gestational age

Ans: A, B, D Factors such as maternal diabetes or multiple gestations place the woman at risk for hydramnios. In addition, there is a discrepancy between fundal height and gestational age, such that a rapid growth of the uterus is noted. Shortness of breath may result from overstretching of the uterus due to the increased amount of amniotic fluid. Often, fetal parts are difficult to palpate and fetal heart rate is difficult to obtain because of the excess fluid present.

After teaching a group of nursing students about variations in newborn head size and appearance, the instructor determines that the teaching was successful when the students identify which of the following as a normal variation? (Select all that apply.) A) Cephalhematoma B) Molding C) Closed fontanels D) Caput succedaneum E) Posterior fontanel diameter 1.5 cm

Ans: A, B, D Normal variations in newborn head size and appearance include cephalhematoma, molding, and caput succedaneum. Microcephaly, closed fontanels, or a posterior fontanel diameter greater than 1 cm are considered abnormal.

After teaching a group of nursing students about tocolytic therapy, the instructor determines that the teaching was successful when they identify which drug as being used for tocolysis? (Select all that apply.) A) Nifedipine B) Terbutaline C) Dinoprostone D) Misoprostol E) Indomethacin

Ans: A, B, E Medications most commonly used for tocolysis include magnesium sulfate (which reduces the muscle's ability to contract), terbutaline (Brethine, a beta-adrenergic), indomethacin (Indocin, a prostaglandin synthetase inhibitor), and nifedipine (Procardia, a calcium channel blocker). These drugs are used "off label": this means they are effective for this purpose but have not been officially tested and developed for this purpose by the FDA. Dinoprostone and misoprostol are used to ripen the cervix.

A nursing instructor is describing common problems associated with preterm birth. When describing the preterm newborn's risk for perinatal asphyxia, the instructor includes which of the following as contributing to the newborn's risk? (Select all that apply.) A) Surfactant deficiency B) Placental deprivation C) Immaturity of the respiratory control centers D) Decreased amounts of brown fat E) Depleted glycogen stores

Ans: A, C Feedback: Preterm newborns are at risk for perinatal asphyxia due to surfactant deficiency, unstable chest wall, immaturity of the respiratory control centers, small respiratory passages, and inability to clear mucus from the airways. Placental deprivation places the postterm newborn at risk for perinatal asphyxia. Decreased amounts of brown fat and depleted glycogen stores place the SGA newborn at risk for problems with thermoregulation.

A nurse is assessing a pregnant woman on a routine checkup. When assessing the woman's gastrointestinal tract, which of the following would the nurse expect to find? (Select all that apply. A) Hyperemic gums B) Increased peristalsis C) Complaints of bloating D) Heartburn E) Nausea

Ans: A, C, D, E Gastrointestinal system changes include hyperemic gums due to estrogen and increased proliferation of blood vessels and circulation to the mouth; slowed peristalsis; acid indigestion and heartburn; bloating and nausea and vomiting.

The nurse notifies the obstetrical team immediately because the nurse suspects that the pregnant woman may be exhibiting signs and symptoms of amniotic fluid embolism. Which findings would the nurse most likely assess? (Select all that apply.) A) Significant difficulty breathing B) Hypertension C) Tachycardia D) Pulmonary edema E) Bleeding with bruising

Ans: A, C, D, E The clinical appearance is varied, but most women report difficulty breathing. Other symptoms include hypotension, cyanosis, seizures, tachycardia, coagulation failure, disseminated intravascular coagulation, pulmonary edema, uterine atony with subsequent hemorrhage, adult respiratory distress syndrome, and cardiac arrest.

The nurse is assessing a newborn and suspects that the newborn was exposed to drugs in utero because the newborn is exhibiting signs of neonatal abstinence syndrome. Which of the following would the nurse expect to assess? (Select all that apply.) A) Tremors B) Diminished sucking C) Regurgitation D) Shrill, high-pitched cry E) Hypothermia F) Frequent sneezing

Ans: A, C, D, F Feedback: Signs and symptoms of neonatal abstinence syndrome include tremors, frantic sucking, regurgitation or projectile vomiting, shrill high-pitched cry, fever, and frequent sneezing.

A nurse is developing a plan of care for a preterm newborn to address the nursing diagnosis of risk for delayed development. Which of the following would the nurse include? (Select all that apply.) A) Clustering care to promote rest B) Positioning newborn in extension C) Using kangaroo care D) Loosely covering the newborn with blankets E) Providing nonnutritive sucking

Ans: A, C, E Feedback: The nurse would focus the plan of care on developmental care, which includes clustering care to promote rest and conserve energy, using flexed positioning to simulate in utero positioning, using kangaroo care to promote skin to skin sensations, swaddling with a blanket to maintain the flexed position, and providing nonnutritive sucking.

A nurse suspects that a pregnant client may be experiencing abruption placenta based on assessment of which of the following? (Select all that apply.) A) Dark red vaginal bleeding B) Insidious onset C) Absence of pain D) Rigid uterus E) Absent fetal heart tones

Ans: A, D, E Feedback: Assessment findings associated with abruption placenta include a sudden onset, with concealed or visible bleeding, dark red bleeding, constant pain or uterine tenderness on palpation, firm to rigid uterine tone, and fetal distress or absent fetal heart tones.

A woman comes to the prenatal clinic suspecting that she is pregnant, and assessment reveals probable signs of pregnancy. Which of the following would be included as part of this assessment? (Select all that apply A) Positive pregnancy test B) Ultrasound visualization of the fetus C) Auscultation of a fetal heart beat D) Ballottement E) Absence of menstruation F) Softening of the cervix

Ans: A, D, F Probable signs of pregnancy include a positive pregnancy test, ballottement, and softening of the cervix (Goodell's sign). Ultrasound visualization of the fetus, auscultation of a fetal heart beat, and palpation of fetal movements are considered positive signs of pregnancy. Absence of menstruation is a presumptive sign of pregnancy.

Just after delivery, a newborn's axillary temperature is 94° C. What action would be most appropriate? A) Assess the newborn's gestational age. B) Rewarm the newborn gradually. C) Observe the newborn every hour. D) Notify the physician if the temperature goes lower.

Ans: B A newborn's temperature is typically maintained at 36.5 to 37.5° C (97.7 to 99.7° F). Since this newborn's temperature is significantly lower, the nurse should institute measures to rewarm the newborn gradually. Assessment of gestational age is completed regardless of the newborn's temperature. Observation would be inappropriate because lack of action may lead to a further lowering of the temperature. The nurse should notify the physician of the newborn's current temperature since it is outside normal parameters.

A woman is receiving magnesium sulfate as part of her treatment for severe preeclampsia. The nurse is monitoring the woman's serum magnesium levels. Which level would the nurse identify as therapeutic? A) 3.3 mEq/L B) 6.1 mEq/L C) 8.4 mEq/L D) 10.8 mEq/L

Ans: B Although exact levels may vary among agencies, serum magnesium levels ranging from 4 to 7 mEq/L are considered therapeutic, whereas levels more than 8 mEq/dL are generally considered toxic.

A woman with gestational hypertension experiences a seizure. Which of the following would be the priority? A) Fluid replacement B) Oxygenation C) Control of hypertension D) Delivery of the fetus

Ans: B As with any seizure, the priority is to clear the airway and maintain adequate oxygenation both to the mother and the fetus. Fluids and control of hypertension are addressed once the airway and oxygenation are maintained. Delivery of fetus is determined once the seizures are controlled and the woman is stable.

Which of the following would lead the nurse to suspect that a postpartum woman is experiencing a problem? A) Elevated white blood cell count B) Acute decrease in hematocrit C) Increased levels of clotting factors D) Pulse rate of 60 beats/minute

Ans: B Despite a decrease in blood volume after birth, hematocrit levels remain relatively stable and may even increase. An acute decrease is not an expected finding. The WBC count remains elevated for the first 4 to 6 days and clotting factors remain elevated for 2 to 3 weeks. Bradycardia (50 to 70 beats per minute) for the first two weeks reflects the decrease in cardiac output.

A woman with preterm labor is receiving magnesium sulfate. Which finding would require the nurse to intervene immediately? A) Respiratory rate of 16 breaths per minute B) Diminished deep tendon reflexes C) Urine output of 45 mL/hour D) Alert level of consciousness

Ans: B Diminished deep tendon reflexes suggest magnesium toxicity, which requires immediate intervention. Additional signs of magnesium toxicity include a respiratory rate less than 12 breaths/minute, urine output less than 30 mL/hour, and a decreased level of consciousness.

A nurse is making a home visit to a postpartum woman who delivered a healthy newborn 4 days ago. The woman's breasts are swollen, hard, and tender to the touch. The nurse documents this finding as which of the following? A) Involution B) Engorgement C) Mastitis D) Engrossment

Ans: B Engorgement is the process of swelling of the breast tissue as a result of an increase in blood and lymph supply as a precursor to lactation (Figure 15.4). Breast engorgement usually peaks in 3 to 5 days postpartum and usually subsides within the next 24 to 36 hours (Chapman, 2011). Engorgement can occur from infrequent feeding or ineffective emptying of the breasts and typically lasts about 24 hours. Breasts increase in vascularity and swell in response to prolactin 2 to 4 days after birth. If engorged, the breasts will be hard and tender to touch. Involution refers to the process of the uterus returning to its prepregnant state. Mastitis refers to an infection of the breasts. Engrossment refers to the bond that develops between the father and the newborn.

A nurse is describing the risks associated with prolonged pregnancies as part of an inservice presentation. Which of the following would the nurse be least likely to incorporate in the discussion as an underlying reason for problems in the fetus? A) Aging of the placenta B) Increased amniotic fluid volume C) Meconium aspiration D) Cord compression

Ans: B Fetal risks associated with a prolonged pregnancy include macrosomia, shoulder dystocia, brachial plexus injuries, low Apgar scores, postmaturity syndrome (loss of subcutaneous fat and muscle and meconium staining), and cephalopelvic disproportion. All of these conditions predispose this fetus to birth trauma or a surgical birth. Uteroplacental insufficiency, meconium aspiration, and intrauterine infection contribute to the increased rate of perinatal deaths (Beacock, 2011). As the placenta ages, its perfusion decreases and it becomes less efficient at delivering oxygen and nutrients to the fetus. Amniotic fluid volume also begins to decline by 40 weeks of gestation, possibly leading to oligohydramnios, subsequently resulting in fetal hypoxia and an increased risk of cord compression because the cushioning effect offered by adequate fluid is no longer present. Hypoxia and oligohydramnios predispose the fetus to aspiration of meconium, which is released by the fetus in response to a hypoxic insult (Caughey & Butler, 2010).

A postpartum client who is bottle feeding her newborn asks, "When should my period return?" Which response by the nurse would be most appropriate? A) "It's difficult to say, but it will probably return in about 2 to 3 weeks." B) "It varies, but you can estimate it returning in about 7 to 9 weeks." C) "You won't have to worry about it returning for at least 3 months." D) "You don't have to worry about that now. It'll be quite a while."

Ans: B For the nonlactating woman, menstruation resumes 7 to 9 weeks after giving birth, with the first cycle being anovulatory. For the lactating woman, menses can return anytime from 2 to 18 months after childbirth.

A woman with placenta previa is being treated with expectant management. The woman and fetus are stable. The nurse is assessing the woman for possible discharge home. Which statement by the woman would suggest to the nurse that home care might be inappropriate? A) "My mother lives next door and can drive me here if necessary." B) "I have a toddler and preschooler at home who need my attention." C) "I know to call my health care provider right away if I start to bleed again." D) "I realize the importance of following the instructions for my care."

Ans: B Having a toddler and preschooler at home needing attention suggest that the woman would have difficulty maintaining bed rest at home. Therefore, expectant management at home may not be appropriate. Expectant management is appropriate if the mother and fetus are both stable, there is no active bleeding, the client has readily available access to reliable transportation, and can comprehend instructions.

Which of the following would the nurse interpret as being least indicative of paternal engrossment? A) Demonstrating pleasure when touching or holding the newborn B) Identifying imperfections in the newborn's appearance C) Being able to distinguish his newborn from others in the nursery D) Showing feelings of pride with the birth of the newborn

Ans: B Identifying imperfections would not be associated with engrossment. Engrossment is characterized by seven behaviors: visual awareness of the newborn, tactile awareness of the newborn, perception of the newborn as perfect, strong attraction to the newborn, awareness of distinct features of the newborn, extreme elation, and increased sense of self-esteem.

The fetus of a woman in labor is determined to be in persistent occiput posterior position. Which of the following would the nurse identify as the priority intervention? A) Position changes B) Pain relief measures C) Immediate cesarean birth D) Oxytocin administration

Ans: B Intense back pain is associated with persistent occiput posterior position. Therefore, a priority is to provide pain relief measures. Position changes that can promote fetal head rotation are important after the nurse institutes pain relief measures. Additionally, the woman's ability to cooperate and participate in these position changes is enhanced when she is experiencing less pain. Immediate cesarean birth is not indicated unless there is evidence of fetal distress. Oxytocin would add to the woman's already high level of pain.

A group of nursing students are reviewing information about methods used for cervical ripening. The students demonstrate understanding of the information when they identify which of the following as a mechanical method? A) Herbal agents B) Laminaria C) Membrane stripping D) Amniotomy

Ans: B Laminaria is a hygroscopic dilator that is used as a mechanical method for cervical ripening. Herbal agents are a nonpharmacologic method. Membrane stripping and amniotomy are considered surgical methods.

A nurse is visiting a postpartum woman who delivered a healthy newborn 5 days ago. Which of the following would the nurse expect to find? A) Bright red discharge B) Pinkish brown discharge C) Deep red mucus-like discharge D) Creamy white discharge

Ans: B Lochia serosa is pinkish brown and is expelled 3 to 10 days postpartum. Lochia rubra is a deep-red mixture of mucus, tissue debris, and blood that occurs for the first 3 to 4 days after birth. Lochia alba is creamy white or light brown and consists of leukocytes, decidual tissue, and reduced fluid content and occurs from days 10 to 14 but can last 3 to 6 weeks postpartum.

During a physical assessment of a newborn, the nurse observes bluish markings across the newborn's lower back. The nurse documents this finding as which of the following? A) Milia B) Mongolian spots C) Stork bites D) Birth trauma

Ans: B Mongolian spots are blue or purple splotches that appear on the lower back and buttocks of newborns. Milia are unopened sebaceous glands frequently found on a newborn's nose. Stork bites are superficial vascular areas found on the nape of the neck and eyelids and between the eyes and upper lip. Birth trauma would be manifested by bruising, swelling, and possible deformity.

A nurse strongly encourages a pregnant client to avoid eating swordfish and tilefish because these fish contain which of the following? A) Excess folic acid, which could increase the risk for neural tube defects B) Mercury, which could harm the developing fetus if eaten in large amounts C) Lactose, which leads to abdominal discomfort, gas, and diarrhea D) Low-quality protein that does not meet the woman's requirements

Ans: B Nearly all fish and shellfish contain traces of mercury and some contain higher levels of mercury that may harm the developing fetus if ingested by pregnant women in large amounts. Among these fish are shark, swordfish, king mackerel, and tilefish. Folic acid is found in dark green vegetables, baked beans, black-eyed peas, citrus fruits, peanuts, and liver. Folic acid supplements are needed to prevent neural tube defects. Women who are lactose intolerant experience abdominal discomfort, gas, and diarrhea if they ingest foods containing lactose. Fish and shellfish are an important part of a healthy diet because they contain high-quality proteins, are low in saturated fat, and contain omega-3 fatty acids.

Assessment of a pregnant woman reveals that she compulsively craves ice. The nurse documents this finding as which of the following? A) Quickening B) Pica C) Ballottement D) Linea nigra

Ans: B Pica refers to the compulsive ingestion of nonfood substances such as ice. Quickening refers to the mother's sensation of fetal movement. Ballottement refers to the feeling of rebound from a floating fetus when an examiner pushes against the woman's cervix during a pelvic examination. Linea nigra refers to the pigmented line that develops in the middle of the woman's abdomen.

A woman who gave birth 24 hours ago tells the nurse, "I've been urinating so much over the past several hours." Which response by the nurse would be most appropriate? A) "You must have an infection, so let me get a urine specimen." B) "Your body is undergoing many changes that cause your bladder to fill quickly." C) "Your uterus is not contracting as quickly as it should." D) "The anesthesia that you received is wearing off and your bladder is working again."

Ans: B Postpartum diuresis occurs as a result of several mechanisms: the large amounts of IV fluids given during labor, a decreasing antidiuretic effect of oxytocin as its level declines, the buildup and retention of extra fluids during pregnancy, and a decreasing production of aldosterone—the hormone that decreases sodium retention and increases urine production. All these factors contribute to rapid filling of the bladder within 12 hours of birth. Diuresis begins within 12 hours after childbirth and continues throughout the first week postpartum. Rapid bladder filling, possible infection, or effects of anesthesia are not involved.

A nurse is teaching new parents about bathing their newborn. The nurse determines that the teaching was successful when the parents state which of the following? A) "We can put a tiny bit of lotion on his skin and then rub it in gently." B) "We should avoid using any kind of baby powder." C) "We need to bathe him at least four to five times a week." D) "We should clean his eyes after washing his face and hair."

Ans: B Powders should not be used because they can be inhaled, causing respiratory distress. If the parents want to use oils and lotions, have them apply a small amount onto their hand first, away from the newborn; this warms the lotion. Then the parents should apply the lotion or oil sparingly. Parents need to be instructed that a bath two or three times weekly is sufficient for the first year because too frequent bathing may dry the skin. The eyes are cleaned first and only with plain water; then the rest of the face is cleaned with plain water.

A group of nursing students are reviewing respiratory system adaptations that occur during the postpartum period. The students demonstrate understanding of the information when they identify which of the following as a postpartum adaptation? A) Continued shortness of breath B) Relief of rib aching C) Diaphragmatic elevation D) Decrease in respiratory rate

Ans: B Respirations usually remain within the normal adult range of 16 to 24 breaths per minute. As the abdominal organs resume their nonpregnant position, the diaphragm returns to its usual position. Anatomic changes in the thoracic cavity and rib cage caused by increasing uterine growth resolve quickly. As a result, discomforts such as shortness of breath and rib aches are relieved.

The nurse would be alert for possible placental abruption during labor when assessment reveals which of the following? A) Macrosomia B) Gestational hypertension C) Gestational diabetes D) Low parity

Ans: B Risk factors for placental abruption include preeclampsia, gestational hypertension, seizure activity, uterine rupture, trauma, smoking, cocaine use, coagulation defects, previous history of abruption, domestic violence, and placental pathology. Macrosomia, gestational diabetes, and low parity are not considered risk factors.

A postpartum woman who has experienced diastasis recti asks the nurse about what to expect related to this condition. Which response by the nurse would be most appropriate? A) "You'll notice that this will fade to silvery lines." B) "Exercise will help to improve the muscles." C) "Expect the color to lighten somewhat." D) "You'll notice that your shoe size will increase."

Ans: B Separation of the rectus abdominis muscles, called diastasis recti, is more common in women who have poor abdominal muscle tone before pregnancy. After birth, muscle tone is diminished and the abdominal muscles are soft and flabby. Specific exercises are necessary to help the woman regain muscle tone. Fortunately, diastasis responds well to exercise, and abdominal muscle tone can be improved. Stretch marks (striae gravidarum) fade to silvery lines. The darkened pigmentation of the abdomen (linea nigra), face (melasma), and nipples gradually fades. Parous women will note a permanent increase in shoe size.

A primiparous client is being seen in the clinic for her first prenatal visit. It is determined that she is 11 weeks pregnant. The nurse develops a teaching plan to educate the client about what she will most likely experience during this period. Which of the following would the nurse include? A) Ankle edema B) Urinary frequency C) Backache D) Hemorrhoids

Ans: B The client is in her first trimester and would most likely experience urinary frequency as the growing uterus presses on the bladder. Ankle edema, backache, and hemorrhoids would be more common during the later stages of pregnancy.

A nurse is teaching a postpartum client and her partner about caring for their newborn's umbilical cord site. Which statement by the parents indicates a need for additional teaching? A) "We can put him in the tub to bathe him once the cord falls off and is healed." B) "The cord stump should change from brown to yellow." C) "Exposing the stump to the air helps it to dry." D) "We need to call the doctor if we notice a funny odor."

Ans: B The cord stump should change color from yellow to brown or black. Therefore the parents need additional teaching if they state the color changes from brown to yellow. Tub baths are avoided until the cord has fallen off and the area is healed. Exposing the stump to the air helps it to dry. The parents should notify their primary care provider if there is any bleeding, redness, drainage, or foul odor from the cord stump.

A father of a newborn tells the nurse, "I may not know everything about being a dad, but I'm going to do the best I can for my son." The nurse interprets this as indicating the father is in which stage of adaptation? A) Expectations B) Transition to mastery C) Reality D) Taking-in

Ans: B The father's statement reflects transition to mastery because he is making a conscious decision to take control and be at the center of the newborn's life regardless of his preparedness. The expectations stage involves preconceptions about how life will be with a newborn. Reality occurs when fathers realize their expectations are not realistic. Taking-in is a phase of maternal adaptation.

Assessment of a newborn's head circumference reveals that it is 34 cm. The nurse would suspect that this newborn's chest circumference would be: A) 30 cm B) 32 cm C) 34 cm D) 36 cm

Ans: B The newborn's chest should be round, symmetric, and 2 to 3 cm smaller than the head circumference. Therefore, this newborn's chest circumference would be 31 to 32 cm to be normal.

The partner of a woman who has given birth to a healthy newborn says to the nurse, "I want to be involved, but I'm not sure that I'm able to care for such a little baby." The nurse interprets this as indicating which of the following stages? A) Expectations B) Reality C) Transition to mastery D) Taking-hold

Ans: B The partner's statement reflects stage 2 (reality), which occurs when fathers or partners realize that their expectations in stage 1 are not realistic. Their feelings change from elation to sadness, ambivalence, jealousy, and frustration. Many wish to be more involved in the newborn's care and yet do not feel prepared to do so. New fathers or partners pass through stage 1 (expectations) with preconceptions about what home life will be like with a newborn. Many men may be unaware of the dramatic changes that can occur when this newborn comes home to live with them. In stage 3 (transition to mastery), the father or partner makes a conscious decision to take control and be at the center of his newborn's life regardless of his preparedness. Taking-hold is a stage of maternal adaptation.

The nurse is auscultating a newborn's heart and places the stethoscope at the point of maximal impulse at which location? A) Just superior to the nipple, at the midsternum B) Lateral to the midclavicular line at the fourth intercostal space C) At the fifth intercostal space to the left of the sternum D) Directly adjacent to the sternum at the second intercostals space

Ans: B The point of maximal impulse (PMI) in a newborn is a lateral to midclavicular line located at the fourth intercostal space.

A primigravida whose labor was initially progressing normally is now experiencing a decrease in the frequency and intensity of her contractions. The nurse would assess the woman for which condition? A) A low-lying placenta B) Fetopelvic disproportion C) Contraction ring D) Uterine bleeding

Ans: B The woman is experiencing dystocia most likely due to hypotonic uterine dysfunction and fetopelvic disproportion associated with a large fetus. A low-lying placenta, contraction ring, or uterine bleeding would not be associated with a change in labor pattern.

After teaching a couple about what to expect with their planned cesarean birth, which statement indicates the need for additional teaching? A) "Holding a pillow against my incision will help me when I cough." B) "I'm going to have to wait a few days before I can start breast-feeding." C) "I guess the nurses will be getting me up and out of bed rather quickly." D) "I'll probably have a tube in my bladder for about 24 hours or so."

Ans: B Typically, breast-feeding is initiated early as soon as possible after birth to promote bonding. The woman may need to use alternate positioning techniques to reduce incisional discomfort. Splinting with pillows helps to reduce the discomfort associated with coughing. Early ambulation is encouraged to prevent respiratory and cardiovascular problems and promote peristalsis. An indwelling urinary catheter is typically inserted to drain the bladder. It usually remains in place for approximately 24 hours.

When assessing several women for possible VBAC, which woman would the nurse identify as being the best candidate? A) One who has undergone a previous myomectomy B) One who had a previous cesarean birth via a low transverse incision C) One who has a history of a contracted pelvis D) One who has a vertical incision from a previous cesarean birth

Ans: B VBAC is an appropriate choice for women who have had a previous cesarean birth with a lower abdominal transverse incision. It is contraindicated in women who have a prior classic uterine incision (vertical), prior transfundal surgery, such as myomectomy, or a contracted pelvis.

A pregnant woman comes to the clinic and tells the nurse that she has been having a whitish vaginal discharge. The nurse suspects vulvovaginal candidiasis, based on which assessment finding? A) Fever B) Vaginal itching C) Urinary frequency D) Incontinence

Ans: B Vaginal secretions become more acidic, white, and thick during pregnancy. Most women experience an increase in a whitish vaginal discharge, called leukorrhea. This is normal except when it is accompanied by itching and irritation, possibly suggesting Candida albicans, a monilial vaginitis, which is a very common occurrence in this glycogen-rich environment. Fever would suggest a more serious infection. Urinary frequency occurs commonly in the first trimester, disappears during the second trimester, and reappears during the third trimester. Incontinence would not be associated with a vulvovaginal candidiasis. Incontinence would require additional evaluation.

The nurse is developing a plan of care for a woman who is pregnant with twins. The nurse includes interventions focusing on which of the following because of the woman's increased risk? A) Oligohydramnios B) Preeclampsia C) Post-term labor D) Chorioamnionitis

Ans: B Women with multiple gestations are at high risk for preeclampsia, preterm labor, hydramnios, hyperemesis gravidarum, anemia, and antepartal hemorrhage. There is no association between multiple gestations and the development of chorioamnionitis.

The nurse is discussing the insulin needs of a primiparous client with diabetes who has been using insulin for the past few years. The nurse informs the client that her insulin needs will increase during pregnancy based on the nurse's understanding that the placenta produces: A) hCG, which increases maternal glucose levels B) hPL, which deceases the effectiveness of insulin C) Estriol, which interferes with insulin crossing the placenta D) Relaxin, which decreases the amount of insulin produced

Ans: B hPL acts as an antagonist to insulin, so the mother must produce more insulin to overcome this resistance. If the mother has diabetes, then her insulin need would most likely increase to meet this demand. hCG does not affect insulin and glucose level. Estrogen, not estriol, is believed to oppose insulin. In addition, insulin does not cross the placenta. Relaxin is not associated with insulin resistance.

The nurse is reviewing the medical record of a newborn born 2 hours ago. The nurse notes that the newborn was delivered at 35 weeks' gestation. The nurse would classify this newborn as which of the following? A) Preterm B) Late preterm C) Full term D) Postterm

Ans: B Feedback: A late preterm infant is one born between 34 to 36 6/7 weeks of gestation. A preterm infant is one born before 37 completed weeks' gestation. A full-term infant is one born between 38 to 41 weeks' gestation. A postterm newborn is one born at 42 weeks' gestation or later.

When describing newborns with birth-weight variations to a group of nursing students, the instructor identifies which variation if the newborn weighs 5.2 lb at any gestational age? A) Small for gestational age B) Low birth weight C) Very low birth weight D) Extremely low birth weight

Ans: B Feedback: A low-birth-weight newborn weighs less than 5.5 lb (2,500 g) but more than 3 lb 5 oz. A very-low-birth-weight newborn would weigh less than 3 lb 5 oz but more than 2 lb 3 oz (1,000 g). An extremely-low-birth-weight newborn weighs less than 2 lb 3 oz (1,000g). A small-for-gestational-age newborn typically weighs less than 5 lb 8 oz (2,500 g) at term.

A newborn has an Apgar score of 6 at 5 minutes. Which of the following is the priority? A) Initiating IV fluid therapy B) Beginning resuscitative measures C) Promoting kangaroo care D) Obtaining a blood culture

Ans: B Feedback: An Apgar score below 7 at 1 or 5 minutes indicates the need for resuscitation. Intravenous fluid therapy and blood cultures may be done once resuscitation is started. Kangaroo care would be appropriate once the newborn is stable.

Which of the following would the nurse include in the plan of care for a newborn receiving phototherapy? A) Keeping the newborn in the supine position B) Covering the newborn's eyes while under the bililights C) Ensuring that the newborn is covered or clothed D) Reducing the amount of fluid intake to 8 ounces daily

Ans: B Feedback: During phototherapy, the newborn's eyes are covered to protect them from the lights. The newborn is turned every 2 hours to expose all areas of the body to the lights and is kept undressed, except for the diaper area, to provide maximum body exposure to the lights. Fluid intake is increased to allow for added fluid, protein, and calories.

The nurse prepares to administer a gavage feeding for a newborn with transient tachypnea based on the understanding that this type of feeding is necessary for which reason? A) Lactase enzymatic activity is not adequate. B) Oxygen demands need to be reduced. C) Renal solute lead must be considered. D) Hyperbilirubinemia is likely to develop.

Ans: B Feedback: For the newborn with transient tachypnea, the newborn's respiratory rate is high, increasing his oxygen demand. Thus, measures are initiated to reduce this demand. Gavage feedings are one way to do so. With transient tachypnea, enzyme activity and kidney function are not affected. This condition typically resolves within 72 hours. The risk for hyperbilirubinemia is not increased.

The nurse prepares to assess a newborn who is considered to be large for gestational age (LGA). Which of the following would the nurse correlate with this gestational age variation? A) Strong, brisk motor skills B) Difficulty in arousing to a quiet alert state C) Birth weight of 7 lb 14 oz D) Wasted appearance of extremities

Ans: B Feedback: LGA newborns typically are more difficult to arouse to a quiet alert state. They have poor motor skills, have a large body that appears plump and full-sized, and usually weigh more than 8 lb 13 oz at term.

Which of the following would not be considered a risk factor for bronchopulmonary dysplasia (chronic lung disease)? A) Preterm birth (less than 32 weeks) B) Female gender C) White race D) Sepsis

Ans: B Feedback: Male gender is more commonly associated with bronchopulmonary dysplasia. Preterm birth of less than 32 weeks' gestation, sepsis, white race, excessive fluid intake during the first few days of life, severe RDS with mechanical ventilation for more than 1 week, and patent ductus arteriosus are all risk factors associated with chronic lung disease in the newborn.

Which of the following, if noted in the maternal history, would the nurse identify as possibly contributing to the birth of an LGA newborn? A) Drug abuse B) Diabetes C) Preeclampsia D) Infection

Ans: B Feedback: Maternal factors that increase the chance of having an LGA newborn include maternal diabetes mellitus or glucose intolerance, multiparity, prior history of a macrosomic infant, postdated gestation, maternal obesity, male fetus, and genetics. Drug abuse is associated with SGA newborns and preterm newborns. A maternal history of preeclampsia and infection would be associated with preterm birth.

Which of the following would the nurse include when teaching a new mother about the difference between pathologic and physiologic jaundice? A) Physiologic jaundice results in kernicterus. B) Pathologic jaundice appears within 24 hours after birth. C) Both are treated with exchange transfusions of maternal O- blood. D) Physiologic jaundice requires transfer to the NICU.

Ans: B Feedback: Pathologic jaundice appears within 24 hours after birth, whereas physiologic jaundice commonly appears around the third to fourth days of life. Kernicterus is more commonly associated with pathologic jaundice. An exchange transfusion is used only if the total serum bilirubin level remains elevated after intensive phototherapy. With this procedure, the newborn's blood is removed and replaced with nonhemolyzed red blood cells from a donor. Physiologic jaundice often is treated at home.

After teaching the parents of a newborn with periventricular hemorrhage about the disorder and treatment, which statement by the parents indicates that the teaching was successful? A) "We'll make sure to cover both of his eyes to protect them." B) "Our newborn could develop a learning disability later on." C) "Once the bleeding ceases, there won't be any more worries." D) "We need to get family members to donate blood for transfusion."

Ans: B Feedback: Periventricular-intraventricular hemorrhage has long-term sequelae such as seizures, hydrocephalus, periventricular leukomalacia, cerebral palsy, learning disabilities, vision or hearing deficits, and mental retardation. Covering the eyes is more appropriate for the newborn receiving phototherapy. The bleeding in the brain can lead to serious long-term effects. Blood transfusions are not used to treat periventricular hemorrhage.

When planning the care of a newborn addicted to cocaine who is experiencing withdrawal, which of the following would be least appropriate to include? A) Wrapping the newborn snugly in a blanket B) Waking the newborn every hour C) Checking the newborn's fontanels D) Offering a pacifier

Ans: B Feedback: Stimuli need to be decreased. Waking the newborn every hour would most likely be too stimulating. Measures such as swaddling the newborn tightly and offering a pacifier help to decrease irritable behaviors. A pacifier also helps to satisfy the newborn's need for nonnutritive sucking. Checking the fontanels provides evidence of hydration.

When developing the plan of care for a newborn with an acquired condition, which of the following would the nurse include to promote participation by the parents? A) Use verbal instructions primarily for explanations B) Assist with decision making process C) Provide personal views about their decisions D) Encourage them to refrain from showing emotions

Ans: B Feedback: To promote parental participation, the nurse should assist them with making decisions about treatment, and support their decisions for the newborn's care. Imposing personal views about their decisions is inappropriate and undermines the nurse-client relationship. In addition, the nurse would assess their ability to cope with the diagnosis, encourage them to verbalize their feelings about the newborn's condition and treatment and educate them about the newborn's condition using written information and pictures to enhance understanding.

Which of the following would be most appropriate for the nurse to do when assisting parents who have experienced the loss of their preterm newborn? A) Avoid using the terms "death" or "dying." B) Provide opportunities for them to hold the newborn. C) Refrain from initiating conversations with the parents. D) Quickly refocus the parents to a more pleasant topic.

Ans: B Feedback: When dealing with grieving parents, nurses should provide them with opportunities to hold the newborn if they desire. In addition, the nurse should provide the parents with as many memories as possible, encouraging them to see, touch, dress, and take pictures of the newborn. These interventions help to validate the parents' sense of loss, relive the experience, and attach significance to the meaning of loss. The nurse should use appropriate terminology, such as "dying," "died," and "death," to help the parents accept the reality of the death. Nurses need to demonstrate empathy and to respect the parents' feelings, responding to them in helpful and supportive ways. Active listening and allowing the parents to vent their frustrations and anger help validate the parents' feelings and facilitate the grieving process.

The nurse is assessing a newborn's eyes. Which of the following would the nurse identify as normal? (Select all that apply.) A) Slow blink response B) Able to track object to midline C) Transient deviation of the eyes D) Involuntary repetitive eye movement E) Absent red reflex

Ans: B, C, D Assessment of the eyes should reveal a rapid blink reflex, ability to track objects to the midline, transient strabismus (deviation or wandering of the eyes independently), searching nystagmus (involuntary repetitive eye movement), and a red reflex.

A nursing student is preparing a class presentation about changes in the various body systems during the postpartum period and their effects. Which of the following would the student include as influencing a postpartum woman's ability to void? (Select all that apply.) A) Use of an opioid anesthetic during labor B) Generalized swelling of the perineum C) Decreased bladder tone from regional anesthesia D) Use of oxytocin to augment labor E) Need for an episiotomy

Ans: B, C, D Many women have difficulty feeling the sensation to void after giving birth if they received an anesthetic block during labor (which inhibits neural functioning of the bladder) or if they received oxytocin to induce or augment their labor (antidiuretic effect). These women will be at risk for incomplete emptying, bladder distention, difficulty voiding, and urinary retention. In addition, urination may be impeded by perineal lacerations; generalized swelling and bruising of the perineum and tissues surrounding the urinary meatus; hematomas; decreased bladder tone as a result of regional anesthesia; and diminished sensation of bladder pressure as a result of swelling, poor bladder tone, and numbing effects of regional anesthesia used during labor.

A nursing student is reviewing an article about preterm premature rupture of membranes. Which of the following would the student expect to find as factor placing a woman at high risk for this condition? (Select all that apply.) A) High body mass index B) Urinary tract infection C) Low socioeconomic status D) Single gestations E) Smoking

Ans: B, C, E High-risk factors associated with preterm PROM include low socioeconomic status, multiple gestation, low body mass index, tobacco use, preterm labor history, placenta previa, abruptio placenta, urinary tract infection, vaginal bleeding at any time in pregnancy, cerclage, and amniocentesis.

A pregnant woman is admitted with premature rupture of the membranes. The nurse is assessing the woman closely for possible infection. Which of the following would lead the nurse to suspect that the woman is developing an infection? (Select all that apply.) A) Fetal bradycardia B) Abdominal tenderness C) Elevated maternal pulse rate D) Decreased C-reactive protein levels E) Cloudy malodorous fluid

Ans: B, C, E Possible signs of infection associated with premature rupture of membranes include elevation of maternal temperature and pulse rate, abdominal/uterine tenderness, fetal tachycardia over 160 bpm, elevated white blood cell count and C-reactive protein levels, and cloudy, foul-smelling amniotic fluid.

A nurse is developing a teaching plan about nutrition for a group of pregnant women. Which of the following would the nurse include in the discussion? (Select all that apply. A) Keep weight gain to 15 lb B) Eat three meals with snacking C) Limit the use of salt in cooking D) Avoid using diuretics E) Participate in physical activity

Ans: B, D, E To promote optimal nutrition, the nurse would recommend gradual and steady weight gain based on the client's prepregnant weight, eating three meals with one or two snacks daily, not restricting the use of salt unless instructed to do so by the health care provider, avoiding the use of diuretics, and participating in reasonable physical activity daily.

A gravida 2 para 1 client in the 10th week of her pregnancy says to the nurse, ìI've never urinated as often as I have for the past three weeks.î Which response would be most appropriate for the nurse to make? A) ìHaving to urinate so often is annoying. I suggest that you watch how much fluid you are drinking and limit it.î B) ìYou shouldn't be urinating this frequently now; it usually stops by the time you're eight weeks pregnant. Is there anything else bothering you?î C) ìBy the time you are 12 weeks pregnant, this frequent urination should no longer be a problem, but it is likely to return toward the end of your pregnancy.î D) ìWomen having their second child generally don't have frequent urination. Are you experiencing any burning sensations?î

Ans: C As the uterus grows, it presses on the urinary bladder, causing the increased frequency of urination during the first trimester. This complaint lessens during the second trimester only to reappear in the third trimester as the fetus begins to descend into the pelvis, causing pressure on the bladder.

A group of nursing students are reviewing information about maternal and paternal adaptations to the birth of a newborn. The nurse observes the parents interacting with their newborn physically and emotionally. The nurse documents this as which of the following? A) Puerperium B) Lactation C) Attachment D) Engrossment

Ans: C Attachment is a formation of a relationship between a parent and her/his newborn through a process of physical and emotional interactions. Puerperium refers to the postpartum period. Lactation refers to the process of milk secretion by the breasts. Engrossment refers to the bond that develops between the father and the newborn.

The nurse encourages the mother of a healthy newborn to put the newborn to the breast immediately after birth for which reason? A) To aid in maturing the newborn's sucking reflex B) To encourage the development of maternal antibodies C) To facilitate maternal-infant bonding D) To enhance the clearing of the newborn's respiratory passages

Ans: C Breast-feeding can be initiated immediately after birth. This immediate mother-newborn contact takes advantage of the newborn's natural alertness and fosters bonding. This contact also reduces maternal bleeding and stabilizes the newborn's temperature, blood glucose level, and respiratory rate. It is not associated with maturing the sucking reflex, encouraging the development of maternal antibodies, or aiding in clearing of the newborn's respiratory passages.

While performing a physical assessment of a newborn boy, the nurse notes diffuse edema of the soft tissues of his scalp that crosses suture lines. The nurse documents this finding as: A) Molding B) Microcephaly C) Caput succedaneum D) Cephalhematoma

Ans: C Caput succedaneum is localized edema on the scalp, a poorly demarcated soft tissue swelling that crosses the suture lines. Molding refers to the elongated shape of the fetal head as it accommodates to the passage through the birth canal. Microcephaly refers to a head circumference that is 2 standard deviations below average or less than 10% of normal parameters for gestational age. Cephalhematoma is a localized effusion of blood beneath the periosteum of the skull.

A nurse is preparing an inservice education program for a group of nurses about dystocia involving problems with the passenger. Which of the following would the nurse most likely include as the most common problem? A) Macrosomia B) Breech presentation C) Persistent occiput posterior position D) Multifetal pregnancy

Ans: C Common problems involving the passenger include occiput posterior position, breech presentation, multifetal pregnancy, excessive size (macrosomia) as it relates to cephalopelvic disproportion (CPD), and structural anomalies. Of these, persistent occiput posterior is the most common malposition, occurring in about 15% of laboring women.

A primipara client gave birth vaginally to a healthy newborn girl 48 hours ago. The nurse palpates the client's fundus, expecting it to be at which location? A) Two fingerbreadths above the umbilicus B) At the level of the umbilicus C) Two fingerbreadths below the umbilicus D) Four fingerbreadths below the umbilicus

Ans: C During the first few days after birth, the uterus typically descends downward from the level of the umbilicus at a rate of 1 cm (1 fingerbreadth) per day so that by day 2, it is about 2 fingerbreadths below the umbilicus.

A woman in her second trimester comes for a follow-up visit and says to the nurse, ìI feel like I'm on an emotional roller-coaster.î Which response by the nurse would be most appropriate? A) How often has this been happening to you? B) Maybe you need some medication to level things out. C) Mood swings are completely normal during pregnancy. D) Have you been experiencing any thoughts of harming yourself?

Ans: C Emotional lability is characteristic throughout most pregnancies. One moment a woman can feel great joy, and within a short time she can feel shock and disbelief. Frequently, pregnant women will start to cry without any apparent cause. Some women feel as though they are riding an emotional roller-coaster. These extremes in emotion can make it difficult for partners and family members to communicate with the pregnant woman without placing blame on themselves for their mood changes. Clear explanations about how common mood swings are during pregnancy are essential.

The nurse is assessing the skin of a newborn and notes a rash on the newborn's face, and chest. The rash consists of small papules and is scattered with no pattern. The nurse interprets this finding as which of the following? A) Harlequin sign B) Nevus flammeus C) Erythema toxicum D) Port wine stain

Ans: C Erythema toxicum (newborn rash) is a benign, idiopathic, generalized, transient rash that occurs in up to 70% of all newborns during the first week of life. It consists of small papules or pustules on the skin resembling flea bites. The rash is common on the face, chest, and back. One of the chief characteristics of this rash is its lack of pattern. It is caused by the newborn's eosinophils reacting to the environment as the immune system matures. Harlequin sign refers to the dilation of blood vessels on only one side of the body, giving the newborn the appearance of wearing a clown suit. It gives a distinct midline demarcation, which is described as pale on the nondependent side and red on the opposite, dependent side. Nevus flammeus or port wine stain is a capillary angioma located directly below the dermis. It is flat with sharp demarcations and is purple-red. This skin lesion is made up of mature capillaries that are congested and dilated.

A woman in labor is experiencing hypotonic uterine dysfunction. Assessment reveals no fetopelvic disproportion. Which group of medications would the nurse expect to administer? A) Sedatives B) Tocolytics C) Oxytocin D) Corticosteroids

Ans: C For hypotonic labor, a uterine stimulant such as oxytocin may be ordered once fetopelvic disproportion is ruled out. Sedatives might be helpful for the woman with hypertonic uterine contractions to promote rest and relaxation. Tocolytics would be ordered to control preterm labor. Corticosteroids may be given to enhance fetal lung maturity for women experiencing preterm labor.

A woman hospitalized with severe preeclampsia is being treated with hydralazine to control blood pressure. Which of the following would the lead the nurse to suspect that the client is having an adverse effect associated with this drug? A) Gastrointestinal bleeding B) Blurred vision C) Tachycardia D) Sweating

Ans: C Hydralazine reduces blood pressure but is associated with adverse effects such as palpitation, tachycardia, headache, anorexia, nausea, vomiting, and diarrhea. It does not cause gastrointestinal bleeding, blurred vision, or sweating. Magnesium sulfate may cause sweating.

Which of the following data on a client's health history would the nurse identify as contributing to the client's risk for an ectopic pregnancy? A) Use of oral contraceptives for 5 years B) Ovarian cyst 2 years ago C) Recurrent pelvic infections D) Heavy, irregular menses

Ans: C In the general population, most cases of ectopic pregnancy are the result of tubal scarring secondary to pelvic inflammatory disease. Oral contraceptives, ovarian cysts, and heavy, irregular menses are not considered risk factors for ectopic pregnancy.

The nurse is assessing a pregnant woman in the second trimester. Which of the following tasks would indicate to the nurse that the client is incorporating the maternal role into her personality? A) The woman demonstrates concern for herself and her fetus as a unit. B) The client identifies what she must give up to assume her new role. C) The woman acknowledges the fetus as a separate entity within her. D) The client demonstrates unconditional acceptance without rejection.

Ans: C Incorporation of the maternal role into her personality indicates acceptance by the pregnant woman. In doing so, the woman becomes able to identify the fetus as a separate individual. Demonstrating concern for herself and her fetus as a unit is associated with introversion and more commonly occurs during the third trimester. Identification of what the mother must give up to assume the new role occurs during the first trimester. Demonstrating unconditional acceptance without rejection occurs during the third trimester.

The nurse develops a teaching plan for a postpartum client and includes teaching about how to perform Kegel exercises. The nurse includes this information for which reason? A) Reduce lochia B) Promote uterine involution C) Improve pelvic floor tone D) Alleviate perineal pain

Ans: C Kegel exercises help to improve pelvic floor tone, strengthen perineal muscles, and promote healing, ultimately helping to prevent urinary incontinence later in life. Kegel exercises have no effect on lochia, involution, or pain.

The nurse completes the initial assessment of a newborn. Which finding would lead the nurse to suspect that the newborn is experiencing difficulty with oxygenation? A) Respiratory rate of 54 breaths/minute B) Abdominal breathing C) Nasal flaring D) Acrocyanosis

Ans: C Nasal flaring is a sign of respiratory difficulty in the newborn. A rate of 54 breaths/minute, diaphragmatic/abdominal breathing, and acrocyanosis are normal findings.

A woman suspecting she is pregnant asks the nurse about which signs would confirm her pregnancy. The nurse would explain that which of the following would confirm the pregnancy? A) Absence of menstrual period B) Abdominal enlargement C) Palpable fetal movement D) Morning sickness

Ans: C Only positive signs of pregnancy would confirm a pregnancy. The positive signs of pregnancy confirm that a fetus is growing in the uterus. Visualizing the fetus by ultrasound, palpating for fetal movements, and hearing a fetal heartbeat are all signs that make the pregnancy a certainty. Absence of menstrual period and morning sickness are presumptive signs, which can be due to conditions other than pregnancy. Abdominal enlargement is a probable sign.

A new mother who is breast-feeding her newborn asks the nurse, "How will I know if my baby is drinking enough?" Which response by the nurse would be most appropriate? A) "If he seems content after feeding, that should be a sign." B) "Make sure he drinks at least 5 minutes on each breast." C) "He should wet between 6 to 12 diapers each day." D) "If his lips are moist, then he's okay."

Ans: C Soaking 6 to 12 diapers a day indicates adequate hydration. Contentedness after feeding is not an indicator for adequate hydration. Typically a newborn wakes up 8 to 12 times per day for feeding. As the infant gets older, the time on the breast increases. Moist mucous membranes help to suggest adequate hydration but this is not the best indicator.

A woman with hyperemesis gravidarum asks the nurse about suggestions to minimize nausea and vomiting. Which suggestion would be most appropriate for the nurse to make? A) "Make sure that anything around your waist is quite snug." B) "Try to eat three large meals a day with less snacking." C) "Drink fluids in between meals rather than with meals." D) "Lie down for about an hour after you eat"

Ans: C Suggestions to minimize nausea and vomiting include avoiding tight waistbands to minimize pressure on the abdomen, eating small frequent meals throughout the day, separating fluids from solids by consuming fluids in between meals; and avoiding lying down or reclining for at least 2 hours after eating.

A nurse is assessing a pregnant woman who has come to the clinic. The woman reports that she feels some heaviness in her thighs since yesterday. The nurse suspects that the woman may be experiencing preterm labor based on which additional assessment finding? A) Dull low backache B) Malodorous vaginal discharge C) Dysuria D) Constipation

Ans: C Symptoms of preterm labor are often subtle and may include change or increase in vaginal discharge with mucus, water, or blood in it; pelvic pressure; low, dull backache; nausea, vomiting or diarrhea, and intestinal cramping with or without diarrhea.

When the nurse is assessing a postpartum client approximately 6 hours after delivery, which finding would warrant further investigation? A) Deep red, fleshy-smelling lochia B) Voiding of 350 cc C) Heart rate of 120 beats/minute D) Profuse sweating

Ans: C Tachycardia in the postpartum woman warrants further investigation. It may indicate hypovolemia, dehydration, or hemorrhage. Deep red, fleshy-smelling lochia is a normal finding 6 hours postpartum. Voiding in small amounts such as less than 150 cc would indicate a problem, but 350 cc would be appropriate. Profuse sweating also is normal during the postpartum period.

Upon entering the room of a client who has had a spontaneous abortion, the nurse observes the client crying. Which of the following responses by the nurse would be most appropriate? A) "Why are you crying?" B) "Will a pill help your pain?" C) "I'm sorry you lost your baby." D) "A baby still wasn't formed in your uterus."

Ans: C Telling the client that the nurse is sorry for the loss acknowledges the loss to the woman, validates her feelings, and brings the loss into reality. Asking why the client is crying is ineffective at this time. Offering a pill for the pain ignores the client's feelings. Telling the client that the baby wasn't formed is inappropriate and discounts any feelings or beliefs that the client has.

After teaching a group of nursing students about the possible causes of spontaneous abortion, the instructor determines that the teaching was successful when the students identify which of the following as the most common cause of first trimester abortions? A) Maternal disease B) Cervical insufficiency C) Fetal genetic abnormalities D) Uterine fibroids

Ans: C The causes of spontaneous abortion are varied and often unknown. The most common cause for first-trimester abortions is fetal genetic abnormalities, usually unrelated to the mother. Chromosomal abnormalities are more likely causes in first trimester and maternal disease is more likely in the second trimester. Those occurring during the second trimester are more likely related to maternal conditions, such as cervical insufficiency, congenital or acquired anomaly of the uterine cavity (uterine septum or fibroids), hypothyroidism, diabetes mellitus, chronic nephritis, use of crack cocaine, inherited and acquired thrombophilias, lupus, polycystic ovary syndrome, severe hypertension and acute infection such as rubella virus, cytomegalovirus, herpes simplex virus, bacterial vaginosis, and toxoplasmosis.

A nurse is assessing a postpartum woman's adjustment to her maternal role. Which of the following would the nurse expect to occur first? A) Reestablishing relationships with others B) Demonstrating increasing confidence in care of the newborn C) Assuming a passive role in meeting her own needs D) Becoming preoccupied with the present

Ans: C The first task of adjusting to the maternal role is the taking-in phase in which the mother demonstrates dependent behaviors and assumes a passive role in meeting her own basic needs. During the taking-hold phase, the mother becomes preoccupied with the present. During the letting-go phase, the mother reestablishes relationships with others and demonstrates increased responsibility and confidence in caring for the newborn.

A woman is at 20 weeks' gestation. The nurse would expect to find the fundus at which of the following? A) Just above the symphysis pubis B) Mid-way between the pubis and umbilicus C) At the level of the umbilicus D) Mid-way between the umbilicus and xiphoid process

Ans: C The uterus, which starts as a pear-shaped organ, becomes ovoid as length increases over width. By 20 weeks' gestation, the fundus, or top of the uterus, is at the level of the umbilicus and measures 20 cm. A monthly measurement of the height of the top of the uterus in centimeters, which corresponds to the number of gestational weeks, is commonly used to date the pregnancy.

A client with hyperemesis gravidarum is admitted to the facility after being cared for at home without success. Which of the following would the nurse expect to include in the client's plan of care? A) Clear liquid diet B) Total parenteral nutrition C) Nothing by mouth D) Administration of labetalol

Ans: C Typically, on admission, the woman with hyperemesis has oral food and fluids withheld for the first 24 to 36 hours to rest the gut and receives parenteral fluids to rehydrate and reduce the symptoms. Once the condition stabilizes, oral intake is gradually increased. Total parenteral nutrition may be used if the client's condition does not improve with several days of bed rest, gut rest, IV fluids, and antiemetics. Labetalol is an antihypertensive agent that may be used to treat gestational hypertension, not hyperemesis.

A woman with a history of crack cocaine abuse is admitted to the labor and birth area. While caring for the client, the nurse notes a sudden onset of fetal bradycardia. Inspection of the abdomen reveals an irregular wall contour. The client also complains of acute abdominal pain that is continuous. Which of the following would the nurse suspect? A) Amniotic fluid embolism B) Shoulder dystocia C) Uterine rupture D) Umbilical cord prolapse

Ans: C Uterine rupture is associated with crack cocaine use, and generally the first and most reliable sign is sudden fetal distress accompanied by acute abdominal pain, vaginal bleeding, hematuria, irregular wall contour, and loss of station in the fetal presenting part. Amniotic fluid embolism often is manifested with a sudden onset of respiratory distress. Shoulder dystocia is noted when continued fetal descent is obstructed after the fetal head is delivered. Umbilical cord prolapse is noted as the protrusion of the cord alongside or ahead of the presenting part of the fetus.

The health care provider orders PGE2 for a woman to help evacuate the uterus following a spontaneous abortion. Which of the following would be most important for the nurse to do? A) Use clean technique to administer the drug. B) Keep the gel cool until ready to use. C) Maintain the client for 1/2 hour after administration. D) Administer intramuscularly into the deltoid area.

Ans: C When PGE2 is ordered, the gel should come to room temperature before administering it. Sterile technique should be used and the client should remain supine for 30 minutes after administration. RhoGAM is administered intramuscularly into the deltoid area.

Which of the following changes in the musculoskeletal system would the nurse mention when teaching a group of pregnant women about the physiologic changes of pregnancy? A) Ligament tightening B) Decreased swayback C) Increased lordosis D) Joint contraction

Ans: C With pregnancy, the woman's center of gravity shifts forward, requiring a realignment of the spinal curvatures. There is an increase in the normal lumbosacral curve (lordosis). Ligaments of the sacroiliac joints and pubis symphysis soften and stretch. Increased swayback and an upper spine extension to compensate for the enlarging abdomen occur. Joint relaxation and increased mobility occur due to the influence of the hormones relaxin and progesterone.

When caring for a mother who has had a cesarean birth, the nurse would expect the client's lochia to be: A) Greater than after a vaginal delivery B) About the same as after a vaginal delivery C) Less than after a vaginal delivery D) Saturated with clots and mucus

Ans: C Women who have had cesarean births tend to have less flow because the uterine debris is removed manually along with delivery of the placenta.

During a vaginal exam, the nurse notes that the cervix has a bluish color. The nurse documents this finding as: A) Hegar's sign B) Goodell's sign C) Chadwick's sign D) Ortolani's sign

Ans: C Bluish coloration of the cervix is termed Chadwick's sign. Hegar's sign refers to the softening of the lower uterine segment or isthmus. Goodell's sign refers to the softening of the cervix. Ortolani's sign is a maneuver done to identify developmental dysplasia of the hip in infants.

When assessing a postterm newborn, which of the following would the nurse correlate with this gestational age variation? A) Moist, supple, plum skin appearance B) Abundant lanugo and vernix C) Thin umbilical cord D) Absence of sole creases

Ans: C Feedback: A postterm newborn typically exhibits a thin umbilical cord; dry, cracked, wrinkled skin; limited vernix and lanugo; and creases covering the entire soles of the feet.

The nurse frequently assesses the respiratory status of a preterm newborn based on the understanding that the newborn is at increased risk for respiratory distress syndrome because of which of the following? A) Inability to clear fluids B) Immature respiratory control center C) Deficiency of surfactant D) Smaller respiratory passages

Ans: C Feedback: A preterm newborn is at increased risk for respiratory distress syndrome (RDS) most commonly because of a surfactant deficiency. Surfactant helps to keep the alveoli open and maintain lung expansion. With a deficiency, the alveoli collapse, predisposing the newborn to RDS. An inability to clear fluids can lead to transient tachypnea. Immature respiratory control centers lead to an increased risk for apnea. Smaller respiratory passages lead to an increased risk for obstruction.

The nurse is assessing a newborn who is large for gestational age. The newborn was born breech. The nurse suspects that the newborn may have experienced trauma to the upper brachial plexus based on which assessment findings? A) Absent grasp reflex B) Hand weakness C) Absent Moro reflex D) Facial asymmetry

Ans: C Feedback: An injury to the upper brachial plexus, or Erb's palsy, is manifested by adduction, pronation, and internal rotation of the affected extremity, absent shoulder movement, absent Moro reflex and positive grasp reflex. An absent grasp reflex and hand weakness is noted with a lower brachial plexus injury. Facial asymmetry is associated with a cranial nerve injury.

The nurse is assessing a preterm newborn's fluid and hydration status. Which of the following would alert the nurse to possible overhydration? A) Decreased urine output B) Tachypnea C) Bulging fontanels D) Elevated temperature

Ans: C Feedback: Bulging fontanels in a preterm newborn suggest overhydration. Sunken fontanels, decreased urine output, and elevated temperature would suggest dehydration.

A nurse is teaching the mother of a newborn diagnosed with galactosemia about dietary restrictions. The nurse determines that the mother has understood the teaching when she identifies which of the following as needing to be restricted? A) Phenylalanine B) Protein C) Lactose D) Iodine

Ans: C Feedback: Lifelong restriction of lactose is required for galactosemia. Phenylalanine is restricted for those with phenylketonuria. Low protein is needed with maple syrup urine disease. Iodine would not be restricted for any inborn error of metabolism

Which of the following would the nurse expect to assess in a newborn who develops sepsis? A) Increased urinary output B) Interest in feeding C) Hypothermia D) Wakefulness

Ans: C Feedback: Manifestations of sepsis are typically nonspecific and may include hypothermia (temperature instability), oliguria or anuria, lack of interest in feeding, and lethargy.

The parents of a preterm newborn being cared for in the neonatal intensive care unit (NICU) are coming to visit for the first time. The newborn is receiving mechanical ventilation and intravenous fluids and medications and is being monitored electronically by various devices. Which action by the nurse would be most appropriate? A) Suggest that the parents stay for just a few minutes to reduce their anxiety. B) Reassure them that their newborn is progressing well. C) Encourage the parents to touch their preterm newborn. D) Discuss the care they will be giving the newborn upon discharge.

Ans: C Feedback: The NICU environment can be overwhelming. Therefore, the nurse should address their reactions and explain all the equipment being used. On entering the NICU, the nurse should encourage the parents to touch, interact, and hold their newborn. Doing so helps to acquaint the parents with their newborn, promotes self-confidence, and fosters parent-newborn attachment. The parents should be allowed to stay for as long as they feel comfortable. Reassurance, although helpful, may be false reassurance at this time. Discussing discharge care can be done later once the newborn's status improves and plans for discharge are initiated.

Assessment of newborn reveals a large protruding tongue, slow reflexes, distended abdomen, poor feeding, hoarse cry, goiter and dry skin. Which of the following would the nurse suspect? A) Phenylketonuria B) Galactosemia C) Congenital hypothyroidism D) Maple syrup urine disease

Ans: C Feedback: The manifestations listed correlate with congenital hypothyroidism. With phenylketonuria, the infant appears normal at birth but by 6 months of age, signs of slow mental development are evident. Vomiting, poor feeding, failure to thrive, overactivity and musty-smelling urine are additional signs. With maple syrup urine disease, signs and symptoms include lethargy, poor feeding, vomiting, weight loss, seizures, shrill cry, shallow respirations, loss of reflexes, and a sweet maple syrup odor to the urine. With galactosemia, manifestations include vomiting, hypoglycemia, hyperbilirubinemia, poor weight gain, cataracts, and frequent infections.

A nurse is teaching a pregnant woman with preterm premature rupture of membranes who is about to be discharged home about caring for herself. Which statement by the woman indicates a need for additional teaching? A) "I need to keep a close eye on how active my baby is each day." B) "I need to call my doctor if my temperature increases." C) "It's okay for my husband and me to have sexual intercourse." D) "I can shower but I shouldn't take a tub bath."

Ans: C Feedback: The woman with preterm premature rupture of membranes should monitor her baby's activity by performing fetal kick counts daily, check her temperature and report any increases to the health care provider, not insert anything into her vagina or vaginal area, such as tampons or vaginal intercourse, and avoid sitting in a tub bath.

After teaching a group of nursing students about risk factors associated with dystocia, the instructor determines that the teaching was successful when the students identify which of the following as increasing the risk? (Select all that apply.) A) Pudendal block anesthetic use B) Multiparity C) Short maternal stature D) Maternal age over 35 E) Breech fetal presentation

Ans: C, D, E According to the American College of Obstetrics and Gynecology (ACOG, 2009a), factors associated with an increased risk for dystocia include epidural analgesia, excessive analgesia, multiple gestation, hydramnios, maternal exhaustion, ineffective maternal pushing technique, occiput posterior position, longer first stage of labor, nulliparity, short maternal stature (less than 5 feet tall), fetal birth weight (more than 8.8 lb), shoulder dystocia, abnormal fetal presentation or position (breech), fetal anomalies (hydrocephalus), maternal age older than 35 years, high caffeine intake, overweight, gestational age more than 41 weeks, chorioamnionitis, ineffective uterine contractions, and high fetal station at complete cervical dilation.

Which assessment finding would lead the nurse to suspect infection as the cause of a client's PROM? A) Yellow-green fluid B) Blue color on Nitrazine testing C) Ferning D) Foul odor

Ans: D A foul odor of the amniotic fluid indicates infection. Yellow-green fluid would suggest meconium. A blue color on Nitrazine testing and ferning indicate the presence of amniotic fluid.

Which finding would indicate to the nurse that a woman's cervix is ripe in preparation for labor induction? A) Posterior position B) Firm C) Closed D) Shortened

Ans: D A ripe cervix is shortened, centered (anterior), softened, and partially dilated. An unripe cervix is long, closed, posterior, and firm.

The nurse is teaching a pregnant woman about recommended weight gain. The woman has a prepregnancy body mass index of 26. The nurse determines that the teaching was successful when the woman states that she should gain no more than which amount during pregnancy? A) 35 to 40 pounds B) 25 to 35 pounds C) 28 to 40 pounds D) 15 to 25 pounds

Ans: D A woman with a body mass index of 26 is considered overweight and should gain no more than 15 to 25 pounds during pregnancy. Women with a body mass index of 18.5 to 24.9 (considered healthy weight) should gain 25 to 35 pounds. A woman with a body mass index less than 18.5 should gain 28 to 40 pounds.

A nurse teaches a postpartum woman about her risk for thromboembolism. Which of the following would the nurse be least likely to include as a factor increasing her risk? A) Increased clotting factors B) Vessel damage C) Immobility D) Increased red blood cell production

Ans: D Clotting factors that increased during pregnancy tend to remain elevated during the early postpartum period. Giving birth stimulates this hypercoagulability state further. As a result, these coagulation factors remain elevated for 2 to 3 weeks postpartum (Silver & Major, 2010). This hypercoagulable state, combined with vessel damage during birth and immobility, places the woman at risk for thromboembolism (blood clots) in the lower extremities and the lungs. Red blood cell production ceases early in the puerperium, which causes mean hemoglobin and hematocrit levels to decrease slightly in the first 24 hours and then rise slowly over the next 2 weeks.

Which of the following findings on a prenatal visit at 10 weeks might lead the nurse to suspect a hydatidiform mole? A) Complaint of frequent mild nausea B) Blood pressure of 120/84 mm Hg C) History of bright red spotting 6 weeks ago D) Fundal height measurement of 18 cm

Ans: D Findings with a hydatidiform mole may include uterine size larger than expected. Mild nausea would be a normal finding at 10 weeks' gestation. Blood pressure of 120/84 would not be associated with hydatidiform mole and depending on the woman's baseline blood pressure may be within acceptable parameters for her. Bright red spotting might suggest a spontaneous abortion.

Which of the following findings would the nurse interpret as suggesting a diagnosis of gestational trophoblastic disease? A) Elevated hCG levels, enlarged abdomen, quickening B) Vaginal bleeding, absence of FHR, decreased hPL levels C) Visible fetal skeleton on ultrasound, absence of quickening, enlarged abdomen D) Gestational hypertension, hyperemesis gravidarum, absence of FHR

Ans: D Gestational trophoblastic disease may be manifested by early development of preeclampsia (gestational hypertension), severe morning sickness due to high hCG levels, and absence of fetal heart rate or activity. There is no fetus, so quickening and evidence of a fetal skeleton would not be seen. The abdominal enlargement is greater than expected for pregnancy dates, but hCG, not hPL, levels are increased.

The nurse is reviewing the laboratory test results of a pregnant client. Which one of the following findings would alert the nurse to the development of HELLP syndrome? A) Hyperglycemia B) Elevated platelet count C) Leukocytosis D) Elevated liver enzymes

Ans: D HELLP is an acronym for hemolysis, elevated liver enzymes, and low platelets. Hyperglycemia or leukocytosis is not a part of this syndrome.

Assessment of a woman in labor who is experiencing hypertonic uterine dysfunction would reveal contractions that are: A) Well coordinated B) Poor in quality C) Rapidly occurring D) Erratic

Ans: D Hypertonic contractions occur when the uterus never fully relaxes between contractions, making the contractions erratic and poorly coordinated because more than one uterine pacemaker is sending signals for contraction. Hypotonic uterine contractions are poor in quality and lack sufficient intensity to dilate and efface the cervix. Contractions of precipitous labor occur rapidly such that labor is completed in less than 3 hours.

A nurse is teaching a pregnant woman at risk for preterm labor about what to do if she experiences signs and symptoms. The nurse determines that the teaching was successful when the woman states that if she experiences any symptoms, she will do which of the following? A) "I'll sit down to rest for 30 minutes." B) "I'll try to move my bowels." C) "I'll lie down with my legs raised." D) "I'll drink several glasses of water."

Ans: D If the woman experiences any signs and symptoms of preterm labor, she should stop what she is doing and rest for 1 hour, empty her bladder, lie down on her side, drink two to three glasses of water, feel her abdomen and note the hardness of the contraction, and call her health care provider and describe the contraction.

The nurse is developing a teaching plan for a client who has decided to bottle feed her newborn. Which of the following would the nurse include in the teaching plan to facilitate suppression of lactation? A) Encouraging the woman to manually express milk B) Suggesting that she take frequent warm showers to soothe her breasts C) Telling her to limit the amount of fluids that she drinks D) Instructing her to apply ice packs to both breasts every other hour

Ans: D If the woman is not breast-feeding, relief measures for engorgement include wearing a tight supportive bra 24 hours daily, applying ice to her breasts for approximately 15 to 20 minutes every other hour, and not stimulating her breasts by squeezing or manually expressing milk. Warm showers enhance the let-down reflex and would be appropriate if the woman was breast-feeding. Limiting fluid intake is inappropriate. Fluid intake is important for all postpartum women, regardless of the feeding method chosen.

After teaching a group of nursing students about the process of involution, the instructor determines that additional teaching is needed when the students identify which of the following as being involved? A) Catabolism B) Muscle fiber contraction C) Epithelial regeneration D) Vasodilation

Ans: D Involution involves three retrogressive Process: contraction of muscle fibers to reduce those previously stretched during pregnancy; catabolism, which reduces enlarged myometrial cells; and regeneration of uterine epithelium from the lower layer of the decidua after the upper layers have been sloughed off and shed during lochial discharge. Vasodilation is not involved.

While talking with a woman in her third trimester, which behavior indicates to the nurse that the woman is learning to give of oneself? A) Showing concern for self and fetus as a unit B) Unconditionally accepting the pregnancy without rejection C) Longing to hold infant D) Questioning ability to become a good mother

Ans: D Learning to give of oneself would be demonstrated when the woman questions her ability to become a good mother to the infant. Showing concern for herself and fetus as a unit reflects the task of ensuring safe passage throughout pregnancy and birth. Unconditionally accepting the pregnancy reflects the task of seeking acceptance of the infant by others. Longing to hold the infant reflects the task of seeking acceptance of self in the maternal role to the infant.

The nurse teaches a primigravida client that lightening occurs about 2 weeks before the onset of labor. The mother will most likely experience which of the following at that time? A) Dysuria B) Dyspnea C) Constipation D) Urinary frequency

Ans: D Lightening refers to the descent of the fetal head into the pelvis and engagement. With this descent, pressure on the diaphragm decreases, easing breathing, but pressure on the bladder increases, leading to urinary frequency. Dysuria might indicate a urinary tract infection. Constipation may occur throughout pregnancy due to decreased peristalsis, but it is unrelated to lightening.

A pregnant client undergoing labor induction is receiving an oxytocin infusion. Which of the following findings would require immediate intervention? A) Fetal heart rate of 150 beats/minute B) Contractions every 2 minutes, lasting 45 seconds C) Uterine resting tone of 14 mm Hg D) Urine output of 20 mL/hour

Ans: D Oxytocin can lead to water intoxication. Therefore, a urine output of 20 mL/hour is below acceptable limits of 30 mL/hour and requires intervention. FHR of 150 beats/minute is within the accepted range of 120 to 160 beats/minute. Contractions should occur every 2 to 3 minutes, lasting 40 to 60 seconds. A uterine resting tone greater than 20 mm Hg would require intervention.

A group of students are reviewing the process of breast milk production. The students demonstrate understanding when they identify which hormone as responsible for milk let-down? A) Prolactin B) Estrogen C) Progesterone D) Oxytocin

Ans: D Oxytocin is released from the posterior pituitary to promote milk let-down. Prolactin levels increase at term with a decrease in estrogen and progesterone; estrogen and progesterone levels decrease after the placenta is delivered. Prolactin is released from the anterior pituitary gland and initiates milk production.

After spontaneous rupture of membranes, the nurse notices a prolapsed cord. The nurse immediately places the woman in which position? A) Supine B) Side-lying C) Sitting D) Knee-chest

Ans: D Pressure on the cord needs to be relieved. Therefore, the nurse would position the woman in a modified Sims, Trendelenburg, or knee-chest position. Supine, side-lying, or sitting would not provide relief of cord compression.

A nursing instructor is teaching a class to a group of students about pregnancy, insulin, and glucose. Which of the following would the instructor least likely include as opposing insulin? A) Prolactin B) Estrogen C) Progesterone D) Cortisol

Ans: D Prolactin, estrogen, and progesterone are all thought to oppose insulin. As a result, glucose is less likely to enter the mother's cells and is more likely to cross over the placenta to the fetus. After the first trimester, hPL from the placenta and steroids (cortisol) from the adrenal cortex act against insulin. hPL acts as an antagonist against maternal insulin, and thus more insulin must be secreted to counteract the increasing levels of hPL and cortisol during the last half of pregnancy.

A nurse is assessing a pregnant woman with gestational hypertension. Which of the following would lead the nurse to suspect that the client has developed severe preeclampsia? A) Urine protein 300 mg/24 hours B) Blood pressure 150/96 mm Hg C) Mild facial edema D) Hyperreflexia

Ans: D Severe preeclampsia is characterized by blood pressure over 160/110 mm Hg, urine protein levels greater than 500 mg/24 hours and hyperreflexia. Mild facial edema is associated with mild preeclampsia.

A postpartum client comes to the clinic for her 6-week postpartum checkup. When assessing the client's cervix, the nurse would expect the external cervical os to appear: A) Shapeless B) Circular C) Triangular D) Slit-like

Ans: D The external cervical os is no longer shaped like a circle but instead appears as a jagged slit-like opening, often described as a "fish mouth."

When assessing a newborn's reflexes, the nurse strokes the newborn's cheek and the newborn turns toward the side that was stroked and begins sucking. The nurse documents which reflex as being positive? A) Palmar grasp reflex B) Tonic neck reflex C) Moro reflex D) Rooting reflex

Ans: D The rooting reflex is elicited by stroking the newborn's cheek. The newborn should turn toward the side that was stroked and should begin to make sucking movements. The palmar grasp reflex is elicited by placing a finger on the newborn's open palm. The baby's hand will close around the finger. Attempting to remove the finger causes the grip to tighten. The tonic neck reflex is elicited by having the newborn lie on the back and turning the head to one side. The arm toward which the baby is facing should extend straight away from the body with the hand partially open, whereas the arm on the side away from the face is flexed and the fist is clenched tightly. Reversing the direction to which the face is turned reverses the position. The Moro reflex is elicited by placing the newborn on his or her back, supporting the upper body weight of the supine newborn by the arms using a lifting motion without lifting the newborn off the surface. The arms are released suddenly and the newborn will throw the arms outward and flex the knees and then the arms return to the chest. The fingers also spread to form a C.

A client who is breast-feeding her newborn tells the nurse, "I notice that when I feed him, I feel fairly strong contraction-like pain. Labor is over. Why am I having contractions now?" Which response by the nurse would be most appropriate? A) "Your uterus is still shrinking in size; that's why you're feeling this pain." B) "Let me check your vaginal discharge just to make sure everything is fine." C) "Your body is responding to the events of labor, just like after a tough workout." D) "The baby's sucking releases a hormone that causes the uterus to contract."

Ans: D The woman is describing afterpains, which are usually stronger during breast-feeding because oxytocin released by the sucking reflex strengthens uterine contractions. Afterpains are associated with uterine involution, but the woman's description strongly correlates with the hormonal events of breast-feeding. All women experience afterpains, but they are more acute in multiparous women secondary to repeated stretching of the uterine muscles.

It is determined that a client's blood Rh is negative and her partner's is positive. To help prevent Rh isoimmunization, the nurse anticipates that the client will receive RhoGAM at which time? A) At 34 weeks' gestation and immediately before discharge B) 24 hours before delivery and 24 hours after delivery C) In the first trimester and within 2 hours of delivery D) At 28 weeks' gestation and again within 72 hours after delivery

Ans: D To prevent isoimmunization, the woman should receive RhoGAM at 28 to 32 weeks gestation and again within 72 hours after delivery.

The nurse administers vitamin K intramuscularly to the newborn based on which of the following rationales? A) Stop Rh sensitization B) Increase erythropoiesis C) Enhance bilirubin breakdown D) Promote blood clotting

Ans: D Vitamin K promotes blood clotting by increasing the synthesis of prothrombin by the liver. RhoGAM prevents Rh sensitization. Erythropoietin stimulates erythropoiesis. Phototherapy enhances bilirubin breakdown.

When performing newborn resuscitation, which action would the nurse do first? A) Intubate with an appropriate-sized endotracheal tube. B) Give chest compressions at a rate of 80 times per minute. C) Administer epinephrine intravenously. D) Suction the mouth and then the nose.

Ans: D Feedback: After placing the newborn's head in a "sniffing" position, the nurse would suction the mouth and then the nose. This is followed by ventilation, circulation (chest compressions), and administration of epinephrine.

A pregnant client in her second trimester has a hemoglobin level of 11 g/dL. The nurse interprets this as indicating which of the following? A) Iron-deficiency anemia B) A multiple gestation pregnancy C) Greater-than-expected weight gain D) Hemodilution of pregnancy

Ans: D Feedback: During pregnancy, the red blood cell count increases along with an increase in plasma volume. However, there is a greater increase in the plasma volume as a result of hormonal factors and sodium and water retention. Thus, the plasma increase exceeds the increase in RBCs, resulting in hemodilution of pregnancy, which is also called physiologic anemia of pregnancy. Changes in maternal iron levels would be more indicative of an iron-deficiency anemia. Although anemia may be present with a multiple gestation, an ultrasound would be a more reliable method of identifying it. Weight gain does not correlate with hemoglobin levels.

The nurse is teaching a group of students about the differences between a full-term newborn and a preterm newborn. The nurse determines that the teaching is effective when the students state that the preterm newborn has: A) Fewer visible blood vessels through the skin B) More subcutaneous fat in the neck and abdomen C) Well-developed flexor muscles in the extremities D) Greater surface area in proportion to weight

Ans: D Feedback: Preterm newborns have large body surface areas compared to weight, which allows an increased transfer of heat from their bodies to the environment. Preterm newborns often have thin transparent skin with numerous visible veins, minimal subcutaneous fat, and poor muscle tone.

When teaching a pregnant client about the physiologic changes of pregnancy, the nurse reviews the effect of pregnancy on glucose metabolism. Which of the following would the nurse include as the underlying reason for the effect? A) Pancreatic function is affected by pregnancy. B) Glucose is utilized more rapidly during a pregnancy. C) The pregnant woman increases her dietary intake. D) Glucose moves through the placenta to assist the fetus.

Ans: D Feedback: The growing fetus has large needs for glucose, amino acids, and lipids, placing demands on maternal glucose stores. During the first half of pregnancy, much of the maternal glucose is diverted to the growing fetus. The pancreas continues to function during pregnancy. However, the placental hormones can affect maternal insulin levels. The demand for glucose by the fetus during pregnancy is high, but it is not necessarily used more rapidly. Placental hormones, not the woman's dietary intake, play a major role in glucose metabolism during pregnancy.

Which of the following would alert the nurse to suspect that a newborn has developed NEC? A) Irritability B) Sunken abdomen C) Clay-colored stools D) Bilious vomiting

Ans: D Feedback: The newborn with NEC may exhibit bilious vomiting with lethargy, abdominal distention and tenderness, and bloody stools.

Which of the following would alert the nurse to suspect that a preterm newborn is in pain? A) Bradycardia B) Oxygen saturation level of 94% C) Decreased muscle tone D) Sudden high-pitched cry

Ans: D Feedback: The nurse should suspect pain if the newborn exhibits a sudden high-pitched cry, oxygen desaturation, tachycardia, and increased muscle tone.

A group of nursing students are reviewing the different types of congenital heart disease in infants. The students demonstrate a need for additional review when they identify which of the following as an example of increased pulmonary blood flow (left-to-right shunting)? A) Atrial septal defect B) Tetralogy of Fallot C) Ventricular septal defect D) Patent ductus arteriosus

B

A woman gives birth to a newborn at 36 weeks' gestation. She tells the nurse, "I'm so glad that my baby isn't premature." Which response by the nurse would be most appropriate? A) "You are lucky to have given birth to a term newborn." B) "We still need to monitor him closely for problems." C) "How do you feel about delivering your baby at 36 weeks?" D) "Your baby is premature and needs monitoring in the NICU."

B

After teaching the parents of a newborn with periventricular hemorrhage about the disorder and treatment, which statement by the parents indicates that the teaching was successful? A) "We'll make sure to cover both of his eyes to protect them." B) "Our newborn could develop a learning disability later on." C) "Once the bleeding ceases, there won't be any more worries." D) "We need to get family members to donate blood for transfusion."

B

The nurse is reviewing the medical record of a newborn born 2 hours ago. The nurse notes that the newborn was delivered at 35 weeks' gestation. The nurse would classify this newborn as which of the following? A) Preterm B) Late preterm C) Full term D) Postterm

B

The nurse prepares to administer a gavage feeding for a newborn with transient tachypnea based on the understanding that this type of feeding is necessary for which reason? A) Lactase enzymatic activity is not adequate. B) Oxygen demands need to be reduced. C) Renal solute lead must be considered. D) Hyperbilirubinemia is likely to develop.

B

When describing newborns with birth-weight variations to a group of nursing students, the instructor identifies which variation if the newborn weighs 5.2 lb at any gestational age? A) Small for gestational age B) Low birth weight C) Very low birth weight D) Extremely low birth weight

B

When developing the plan of care for a newborn with an acquired condition, which of the following would the nurse include to promote participation by the parents? A) Use verbal instructions primarily for explanations B) Assist with decision making process C) Provide personal views about their decisions D) Encourage them to refrain from showing emotions

B

When planning the care of a newborn addicted to cocaine who is experiencing withdrawal, which of the following would be least appropriate to include? A) Wrapping the newborn snugly in a blanket B) Waking the newborn every hour C) Checking the newborn's fontanels D) Offering a pacifier

B

Which of the following would be most appropriate for the nurse to do when assisting parents who have experienced the loss of their preterm newborn? A) Avoid using the terms "death" or "dying." B) Provide opportunities for them to hold the newborn. C) Refrain from initiating conversations with the parents. D) Quickly refocus the parents to a more pleasant topic.

B

Which of the following would not be considered a risk factor for bronchopulmonary dysplasia (chronic lung disease)? A) Preterm birth (less than 32 weeks) B) Female gender C) White race D) Sepsis

B

Which of the following would the nurse include in the plan of care for a newborn receiving phototherapy? A) Keeping the newborn in the supine position B) Covering the newborn's eyes while under the bililights C) Ensuring that the newborn is covered or clothed D) Reducing the amount of fluid intake to 8 ounces daily

B

Which of the following would the nurse include when teaching a new mother about the difference between pathologic and physiologic jaundice? A) Physiologic jaundice results in kernicterus. B) Pathologic jaundice appears within 24 hours after birth. C) Both are treated with exchange transfusions of maternal O- blood. D) Physiologic jaundice requires transfer to the NICU.

B

Which of the following, if noted in the maternal history, would the nurse identify as possibly contributing to the birth of an LGA newborn? A) Drug abuse B) Diabetes C) Preeclampsia D) Infection

B

At 17 weeks pregnant, a mother asks the nurse questions about the development of her baby. The mother states that it may be too early to visualize any body structures via ultrasound. The nurse's best response in relation to fetal development at 17 weeks is: a) Myelination of the spinal cord has occurred. b) Differentiation of hard and soft palate can be seen. c) The earlobes are soft with little cartilage. d) Hard tissue (enamel) for teeth has developed.

B Differentiation of hard and soft palate are structures developed by 16 weeks gestation. Myelination of the spinal cord begins at 20 weeks gestation. Soft earlobes with little cartilage develop at 36 weeks gestation. Teeth form hard tissue (enamel) at 18 weeks gestation.

A pregnant client is concerned about a blow to the abdomen if she continues to play basketball during her pregnancy. The nurse's response is based upon her knowledge of which of the following facts concerning amniotic fluid? a) The total amount of amniotic fluid during pregnancy is 300 mL. b) Amniotic fluid functions as a cushion to protect against mechanical injury. c) The fetus does not contribute to the production of amniotic fluid. d) Amniotic fluid is slightly acidic.

B During pregnancy, the amniotic fluid protects against injury. After 20 weeks of pregnancy, fluid volume ranges from 700-1000 mL. Some of the amniotic fluid is contributed by the fetus's excreting urine. Amniotic fluid is slightly alkaline.

A group of nursing students are preparing a presentation for their class about measures to prevent toxoplasmosis. Which of the following would the students be least likely to include? Select all that apply. A) Washing raw fruits and vegetables before eating them B) Cooking all meat to an internal temperature of 140° F C) Wearing gardening gloves when working in the soil D) Avoiding contact with a cat's litter box.

B Feedback: Meats should be cooked to an internal temperature of 160° F. Other measures to prevent toxoplasmosis include peeling or thoroughly washing all raw fruits and vegetables before eating them, wearing gardening gloves when in contact with outdoor soil, and avoiding the emptying or cleaning of a cat's litter box.

Identify a complication of the small-for-gestational-age newborn. a)Hyperglycemia b)Cognitive difficulties c)Leukocytosis d)Hyperthermia

B SGA newborns often subsequently exhibit learning disabilities. The disabilities are characterized by hyperactivity, short attention span, and poor fine motor coordination. Some hearing loss and speech defects also occur. The SGA newborn does not exhibit symptoms of high blood sugars, increased temperatures, and high white blood cell counts.

The preterm newborn is experiencing vomiting, diarrhea, weight loss, irritability, tremors, and tachypnea. What is the best explanation for these symptoms? a)Traumatic birth b)Maternal substance abuse c)Sepsis d)Gestational diabetes

B The severity of withdrawal that an infant experiences can be assessed by using a scoring system such as the Finnegan scale. This scale is based on observations and measurement of the responses to neonatal abstinence from substances. It evaluates the infant on potentially life-threatening signs such as vomiting, diarrhea, weight loss, irritability, tremors, and tachypnea.

Assessment of a newborn reveals uneven gluteal (buttocks) skin creases and a "clunk" when Ortolani's maneuver is performed. Which of the following would the nurse suspect? A) Slipping of the periosteal joint B) Developmental hip dysplasia C) Normal newborn variation D) Overriding of the pelvic bone

B A "clunk" indicates the femoral head hitting the acetabulum as the head reenters the area. This, along with uneven gluteal creases, suggests developmental hip dysplasia. These findings are not a normal variation and are not associated with slipping of the periosteal joint or overriding of the pelvic bone.

The parents of a newborn become concerned when they notice that their baby seems to stop breathing for a few seconds. After confirming the parents' findings by observing the newborn, which of the following actions would be most appropriate? A) Notify the health care provider immediately. B) Assess the newborn for signs of respiratory distress. C) Reassure the parents that this is an expected pattern. D) Tell the parents not to worry since his color is fine.

B Although periods of apnea of less than 20 seconds can occur, the nurse needs to gather additional information about the newborn's respiratory status to determine if this finding is indicative of a developing problem. Therefore, the nurse would need to assess for signs of respiratory distress. Once this information is obtained, then the nurse can notify the health care provider or explain that this finding is an expected one. However, it would be inappropriate to tell the parents not to worry, because additional information is needed. Also, telling them not to worry ignores their feelings and is not therapeutic.

The nurse prepares to assess a newborn who is considered to be large for gestational age (LGA). Which of the following would the nurse correlate with this gestational age variation? A) Strong, brisk motor skills B) Difficulty in arousing to a quiet alert state C) Birthweight of 7 lb 14 oz D) Wasted appearance of extremities

B LGA newborns typically are more difficult to arouse to a quiet alert state. They have poor motor skills, have a large body that appears plump and full-sized, and usually weigh more than 8 lb 13 oz at term.

Which of the following, if noted in the maternal history, would the nurse identify as possibly contributing to the birth of a LGA newborn? A) Drug abuse B) Diabetes C) Preeclampsia D) Infection

B Maternal factors that increase the chance of having an LGA newborn include maternal diabetes mellitus or glucose intolerance, multiparity, prior history of a macrosomic infant, postdates gestation, maternal obesity, male fetus, and genetics. Drug abuse is associated with SGA newborns and preterm newborns. A maternal history of preeclampsia and infection would be associated with preterm birth.

A new mother is changing the diaper of her 20-hour-old newborn and asks why the stool is almost black. Which response by the nurse would be most appropriate? A) "You probably took iron during your pregnancy." B) "This is meconium stool, normal for a newborn." C) "I'll take a sample and check it for possible bleeding." D) "This is unusual and I need to report this."

B Meconium is greenish-black and tarry and usually passed within 12 to 24 hours of birth. This is a normal finding. Iron can cause stool to turn black, but this would not be the case here. The stool is a normal occurrence and does not need to be checked for blood or reported.

Assessment of a newborn reveals rhythmic spontaneous movements. The nurse interprets this as indicating: A) Habituation B) Motor maturity C) Orientation D) Social behaviors

B Motor maturity is evidenced by rhythmic, spontaneous movements. Habituation is manifested by the newborn's ability to respond to the environment appropriately. Orientation involves the newborn's response to new stimuli, such as turning the head to a sound. Social behaviors involve cuddling and snuggling into the arms of a parent.

The nurse is teaching a group of students about the similarities and differences between newborn skin and adult skin. Which statement by the group indicates that additional teaching is needed? A) The newborn's skin and that of an adult are similar in thickness. B) The newborn's sweat glands function fully, just like those of an adult. C) Skin development in the newborn is not complete at birth. D) The newborn has fewer fibrils connecting the dermis and epidermis

B The newborn has sweat glands, like an adult, but full adult functioning is not present until the second or third year of life. The newborn and adult epidermis is similar in thickness and lipid composition, but skin development is not complete at birth. Fewer fibrils connect the dermis and epidermis in the newborn when compared with the adult.

When making a home visit, the nurse observes a newborn sleeping on his back in a bassinet. In one corner of the bassinet is a soft stuffed animal and at the other end is a bulb syringe. The nurse determines that the mother needs additional teaching because of which of the following? A) The newborn should not be sleeping on his back. B) Stuffed animals should not be in areas where infants sleep. C) The bulb syringe should not be kept in the bassinet. D) This newborn should be sleeping in a crib.

B The nurse should instruct the mother to remove all fluffy bedding, quilts, stuffed animals, and pillows from the crib to prevent suffocation. Newborns and infants should be placed on their backs to sleep. Having the bulb syringe nearby in the bassinet is appropriate. Although a crib is the safest sleeping location, a bassinet is appropriate initially.

After teaching a class about hepatic system adaptations after birth, the instructor determines that the teaching was successful when the class identifies which of the following as the process of changing bilirubin from a fat-soluble product to a water-soluble product? A) Hemolysis B) Conjugation C) Jaundice D) Hyperbilirubinemia

B The process in which bilirubin is changed from a fat-soluble product to a water-soluble product is called conjugation. Hemolysis involves the breakdown of blood cells. In the newborn, hemolysis of the red blood cells is the principal source of bilirubin. Jaundice is the manifestation of increased bilirubin in the bloodstream. Hyperbilirubinemia refers to the increased level of bilirubin in the blood.

The nurse places a warmed blanket on the scale when weighing a newborn to minimize heat loss via which mechanism? A) Evaporation B) Conduction C) Convection D) Radiation

B Using a warmed cloth diaper or blanket to cover any cold surface, such as a scale, that touches a newborn directly helps to prevent heat loss through conduction. Drying a newborn and promptly changing wet linens, clothes, or diapers help reduce heat loss via evaporation. Keeping the newborn out of a direct cool draft, working inside an isolette as much as possible, and minimizing the opening of portholes help prevent heat loss via convection. Keeping cribs and isolettes away from outside walls, cold windows, and air conditioners and using radiant warmers while transporting newborns and performing procedures will help reduce heat loss via radiation.

Which of the following would be most appropriate for the nurse to do when assisting parents who have experienced the loss of their preterm newborn? A) Avoid using the terms "death" or "dying." B) Provide opportunities for them to hold the newborn. C) Refrain from initiating conversations with the parents. D) Quickly refocus the parents to a more pleasant topic.

B When dealing with grieving parents, nurses should provide them with opportunities to hold the newborn if they desire. In addition, the nurse should provide the parents with as many memories as possible, encouraging them to see, touch, dress, and take pictures of the newborn. These interventions help to validate the parents' sense of loss, relive the experience, and attach significance to the meaning of loss. The nurse should use appropriate terminology, such as "dying," "died," and "death," to help the parents accept the reality of the death. Nurses need to demonstrate empathy and to respect the parents' feelings, responding to them in helpful and supportive ways. Active listening and allowing the parents to vent their frustrations and anger help validate the parents' feelings and facilitate the grieving process.

The nurse is assessing a newborn of a woman who is suspected of abusing alcohol. Which newborn finding would provide additional evidence to support this suspicion? A) Wide large eyes B) Thin upper lip C) Protruding jaw D) Elongated nose

B) Thin upper lip Newborn characteristics suggesting fetal alcohol spectrum disorder include thin upper lip, small head circumference, small eyes, receding jaw, and short nose. Other features include a low nasal bridge, short palpebral fissures, flat midface, epicanthal folds, and minor ear abnormalities.

The nurse is assessing the newborn of a mother who had gestational diabetes. Which of the following would the nurse expect to find? (Select all that apply.) A) Pale skin color B) Buffalo hump C) Distended upper abdomen D) Excessive subcutaneous fat E) Long slender neck

B, C, D

A prenatal client at 30 weeks gestation is scheduled for a nonstress test (NST) and asks the nurse, "What is this test for?" The nurse correctly responds that the test is used to determine which of the following? Select all that apply. a)Fetal lung maturity b)Adequate fetal oxygenation c)Accelerations of fetal heart rate d)Fetal well-being

B, C, D An NST documents fetal well-being by measuring fetal oxygenation and fetal heart rate accelerations, but not fetal lung maturity.

A newborn is diagnosed with meconium aspiration syndrome. When assessing this newborn, which of the following would the nurse expect to find? (Select all that apply.) A) Pigeon chest B) Prolonged tachypnea C) Intercostal retractions D) High blood pH level E) Coarse crackles on auscultation

B, C, E

A neonate born to a mother who was abusing heroin is exhibiting signs and symptoms of withdrawal. Which of the following would the nurse assess? (Select all that apply.) A) Low whimpering cry B) Hypertonicity C) Lethargy D) Excessive sneezing E) Overly vigorous sucking F) Tremors

B, D, F Feedback: Signs and symptoms of withdrawal, or neonatal abstinence syndrome, include: irritability, hypertonicity, excessive and often high-pitched crying, vomiting, diarrhea, feeding disturbances, respiratory distress, disturbed sleeping, excessive sneezing and yawning, nasal stuffiness, diaphoresis, fever, poor sucking, tremors, and seizures.

A client who is HIV-positive is in her second trimester and remains asymptomatic. She voices concern about her newborn's risk for the infection. Which statement by the nurse would be most appropriate? A. "Antibodies cross the placenta and provide immunity to the newborn." B. "Antiretroviral medications are available to help reduce the risk of transmission." C. "You'll probably have a cesarean birth to prevent exposing your newborn." D. "Wait until after the infant is born, and then something can be done."

B. "Antiretroviral medications are available to help reduce the risk of transmission."

After teaching a pregnant woman with iron deficiency anemia about her prescribed iron supplement, which statement indicates successful teaching? A. "I'll call the primary care provider if my stool is black and tarry." B. "I need to eat foods high in fiber." C. "I should avoid drinking orange juice." D. "I should take my iron with milk."

B. "I need to eat foods high in fiber." Iron supplements can lead to constipation, so the woman needs to increase her intake of fluids and high-fiber foods. Milk inhibits absorption and should be discouraged. Vitamin C-containing fluids such as orange juice are encouraged because they promote absorption. Ideally the woman should take the iron on an empty stomach to improve absorption, but many women cannot tolerate the gastrointestinal discomfort it causes. In such cases, the woman should take it with meals. Iron typically causes the stool to become black and tarry; there is no need for the woman to notify her doctor.

After teaching a couple about what to expect with their planned cesarean birth, which statement indicates the need for additional teaching? A. "Holding a pillow against my incision will help me when I cough." B. "I'm going to have to wait a few days before I can start breastfeeding." C. "I guess the nurses will be getting me up and out of bed rather quickly." D. "I'll probably have a tube in my bladder for about 24 hours or so."

B. "I'm going to have to wait a few days before I can start breastfeeding." explanation: Typically, breastfeeding is initiated early as soon as possible after birth to promote bonding. The woman may need to use alternate positioning techniques to reduce incisional discomfort. Splinting with pillows helps to reduce the discomfort associated with coughing. Early ambulation is encouraged to prevent respiratory and cardiovascular problems and promote peristalsis. An indwelling urinary catheter is typically inserted to drain the bladder. It usually remains in place for approximately 24 hours.

The nurse would be alert for possible placental abruption during labor when assessment reveals which finding? A. macrosomia B. gestational hypertension C. gestational diabetes D. low parity

B. gestational hypertension explanation: Risk factors for placental abruption include preeclampsia, gestational hypertension, seizure activity, uterine rupture, trauma, smoking, cocaine use, coagulation defects, previous history of abruption, domestic violence, and placental pathology. Macrosomia, gestational diabetes, and low parity are not considered risk factors.

When assessing several women for possible VBAC, which woman would the nurse identify as being the best candidate? A. one who has undergone a previous myomectomy B. one who had a previous cesarean birth via a low transverse incision C. one who has a history of a contracted pelvis D. one who has a vertical incision from a previous cesarean birth

B. one who had a previous cesarean birth via a low transverse incision explanation: VBAC is an appropriate choice for women who have had a previous cesarean birth with a lower abdominal transverse incision. It is contraindicated in women who have a prior classic uterine incision (vertical), prior transfundal surgery, such as myomectomy, or a contracted pelvis.

A woman with gestational hypertension experiences a seizure. Which intervention would the nurse identify as the priority? A. fluid replacement B. oxygenation C. control of hypertension D. birth of the fetus

B. oxygenation explanation: As with any seizure, the priority is to clear the airway and maintain adequate oxygenation both to the mother and the fetus. Fluids and control of hypertension are addressed once the airway and oxygenation are maintained. Delivery of fetus is determined once the seizures are controlled and the woman is stable.

A group of nursing students are reviewing the literature in preparation for a class presentation on newborn pain prevention and management. Which of the following would the students be most likely to find about this topic? A) Newborn pain is frequently recognized and treated B) Newborns rarely experience pain with procedures C) Pain is frequently mistaken for irritability or agitation D) Newborns may be less sensitive to pain than adult.

C

A group of pregnant women are discussing high-risk newborn conditions as part of a prenatal class. When describing the complications that can occur in these newborns to the group, which would the nurse include as being at lowest risk? A) Small-for-gestational-age (SGA. newborns B) Large-for-gestational-age (LGA. newborns C) Appropriate-for-gestational-age (AGA. newborns D) Low-birth-weight newborns

C

A group of students are reviewing information about the effects of substances on the newborn. The students demonstrate understanding of the information when they identify which drug as not being associated with teratogenic effects on the fetus? A) Alcohol B) Nicotine C) Marijuana D) Cocaine

C

A newborn is suspected of developing persistent pulmonary hypertension. The nurse would expect to prepare the newborn for which of the following to confirm the suspicion? A) Chest x-ray B) Blood cultures C) Echocardiogram D) Stool for occult blood

C

A newborn is suspected of having fetal alcohol syndrome. Which of the following would the nurse expect to assess? A) Bradypnea B) Hydrocephaly C) Flattened maxilla D) Hypoactivity

C

A nurse is developing a plan of care for a preterm infant experiencing respiratory distress. Which of the following would the nurse be least likely to include in this plan? A) Stimulate the infant with frequent handling. B) Keep the newborn in a warmed isolette. C) Administer oxygen using a oxygen hood. D) Give gavage or continous tube feedings.

C

A nurse is teaching the mother of a newborn diagnosed with galactosemia about dietary restrictions. The nurse determines that the mother has understood the teaching when she identifies which of the following as needing to be restricted? A) Phenylalanine B) Protein C) Lactose D) Iodine

C

A nursing student is preparing a presentation for the class on clubfoot. The student determines that the presentation was successful when the class states which of the following? A) Clubfoot is a common genetic disorder. B) The condition affects girls more often than boys. C) The exact cause of clubfoot is not known. D) The intrinsic form can be manually reduced.

C

Assessment of newborn reveals a large protruding tongue, slow reflexes, distended abdomen, poor feeding, hoarse cry, goiter and dry skin. Which of the following would the nurse suspect? A) Phenylketonuria B) Galactosemia C) Congenital hypothyroidism D) Maple syrup urine disease

C

The nurse frequently assesses the respiratory status of a preterm newborn based on the understanding that the newborn is at increased risk for respiratory distress syndrome because of which of the following? A) Inability to clear fluids B) Immature respiratory control center C) Deficiency of surfactant D) Smaller respiratory passages

C

The nurse is assessing a newborn who is large for gestational age. The newborn was born breech. The nurse suspects that the newborn may have experienced trauma to the upper brachial plexus based on which assessment findings? A) Absent grasp reflex B) Hand weakness C) Absent Moro reflex D) Facial asymmetry

C

The nurse is assessing a preterm newborn's fluid and hydration status. Which of the following would alert the nurse to possible overhydration? A) Decreased urine output B) Tachypnea C) Bulging fontanels D) Elevated temperature

C

The parents of a preterm newborn being cared for in the neonatal intensive care unit (NICU. are coming to visit for the first time. The newborn is receiving mechanical ventilation and intravenous fluids and medications and is being monitored electronically by various devices. Which action by the nurse would be most appropriate? A) Suggest that the parents stay for just a few minutes to reduce their anxiety. B) Reassure them that their newborn is progressing well. C) Encourage the parents to touch their preterm newborn. D) Discuss the care they will be giving the newborn upon discharge.

C

When assessing a postterm newborn, which of the following would the nurse correlate with this gestational age variation? A) Moist, supple, plum skin appearance B) Abundant lanugo and vernix C) Thin umbilical cord D) Absence of sole creases

C

Which of the following would the nurse expect to assess in a newborn who develops sepsis? A) Increased urinary output B) Interest in feeding C) Hypothermia D) Wakefulness

C

A 28-week-gestation newborn experienced birth asphyxia at the time of delivery. What is a long-term complication of birth asphyxia? a)Necrotizing enterocolitis b)Retinopathy of prematurity c)Intraventricular hemorrhage d)Anemia of prematurity

C Birth asphyxia will cause an insult to the brain, and more often than not will cause a bleed or intraventricular hemorrhage. Birth asphyxia is not directly correlated with NEC, retinopathy of prematurity, or anemia of prematurity. These are common for the premature infant, but not necessarily birth asphyxia.

The physician orders an ultrasound for a prenatal client prior to an amniocentesis. The nurse explains to the client that the purpose of the ultrasound is to: a)Determine the gestational sac volume. b)Measure the fetus's crown-rump length. c)Locate the placenta. d)Measure the fetus's biparietal diameter.

C During an amniocentesis, the physician scans the uterus using ultrasound to identify the fetal and placental positions and to identify adequate pockets of amniotic fluid. Determination of the gestational sac volume, measuring the crown-rump length, and measuring the biparietal diameter are aspects of assessing fetal well-being (biophysical profile, or BPP), and may or may not be done prior to the amniocentesis, depending on gestational age.

A prenatal client at 35 weeks gestation is scheduled for an amniocentesis to determine fetal lung maturity. The nurse expects the lecithin/sphingomyelin (L/S) ratio to be: a)0.5:1 b)1:1 c)2:1 d)3:1

C Early in pregnancy, the sphingomyelin concentration in amniotic fluid is greater than the concentration of lecithin, and so the L/S ratio is low (lecithin levels are low and sphingomyelin levels are high). At about 32 weeks gestation, sphingomyelin levels begin to fall and the amount of lecithin begins to increase. By 35 weeks gestation, an L/S ratio of 2:1 (also reported as 2.0) is usually achieved in the normal fetus.

After teaching a pregnant woman with iron deficiency anemia about her prescribed iron supplement, which statement indicates successful teaching? A) "I should take my iron with milk." B) "I should avoid drinking orange juice." C) "I need to eat foods high in fiber." D) "I'll call the doctor if my stool is black and tarry."

C Feedback: Iron supplements can lead to constipation, so the woman needs to increase her intake of fluids and high-fiber foods. Milk inhibits absorption and should be discouraged. Vitamin C-containing fluids such as orange juice are encouraged because they promote absorption. Ideally the woman should take the iron on an empty stomach to improve absorption, but many women cannot tolerate the gastrointestinal discomfort it causes. In such cases, the woman should take it with meals. Iron typically causes the stool to become black and tarry; there is no need for the woman to notify her doctor.

A client states that she had a spontaneous abortion 12 months ago. The client asks if her hormones may have contributed to the loss of the pregnancy. The nurse's response is based upon her knowledge of which of the following facts? a) Implantation occurs when progesterone levels are low. b) hCG reaches a maximum level at 4 weeks gestation. c) Progesterone decreases the contractility of the uterus. d) Progesterone is only produced by the corpus luteum during pregnancy.

C Progesterone decreases the contractility of the uterus, thus preventing uterine contractions that might cause spontaneous abortion. Progesterone must be present in high levels for implantation to occur. After 10 weeks, the placenta takes over the production of progesterone. hCG reaches its maximum level at 50-70 days gestation.

The nurse is preparing an educational workshop on fetal development. Which statement by the student would require the nurse to explain further? a) "True knots are usually associated with a cord that is too long." b) "The average cord length at term is 22 inches." c) "The umbilical cord normally contains two veins and one artery." d) "The high blood volume and Wharton's jelly content of the umbilical cord prevents compression of the cord."

C Umbilical cords appear twisted or spiraled. This is most likely caused by fetal movement. A true knot in the umbilical cord rarely occurs; if it does, the cord is usually long. More common are so-called false knots, caused by the folding of cord vessels. A nuchal cord is said to exist when the umbilical cord encircles the fetal neck. A normal umbilical cord contains one large vein and two smaller arteries. A specialized connective tissue known as Wharton's jelly surrounds the blood vessels in the umbilical cord. This tissue, plus the high blood volume pulsating through the vessels, prevents compression of the umbilical cord in utero.

When assessing a postterm newborn, which of the following would the nurse correlate with this gestational age variation? A) Moist, supple, plum skin appearance B) Abundant lanugo and vernix C) Thin umbilical cord D) Absence of sole creases

C A postterm newborn typically exhibits a thin umbilical cord; dry, cracked, wrinkled skin; limited vernix and lanugo; and creases covering the entire soles of the feet.

The nurse frequently assesses the respiratory status of a preterm newborn based on the understanding that the newborn is at increased risk for respiratory distress syndrome because of which of the following? A) Inability to clear fluids B) Immature respiratory control center C) Deficiency of surfactant D) Smaller respiratory passages

C A preterm newborn is at increased risk for respiratory distress syndrome (RDS) because of a surfactant deficiency. Surfactant helps to keep the alveoli open and maintain lung expansion. With a deficiency, the alveoli collapse, predisposing the newborn to RDS. An inability to clear fluids can lead to transient tachypnea. Immature respiratory control centers lead to an increased risk for apnea. Smaller respiratory passages led to an increased risk for obstruction.

When explaining how a newborn adapts to extrauterine life, the nurse would describe which body systems as undergoing the most rapid changes? A) Gastrointestinal and hepatic B) Urinary and hematologic C) Respiratory and cardiovascular D) Neurological and integumentary

C Although all the body systems of the newborn undergo changes, respiratory gas exchange along with circulatory modifications must occur immediately to sustain extrauterine life.

When describing the complications that can occur in newborns to a group of pregnant women, which would the nurse include as being at lowest risk? A) Small-for-gestational-age newborns B) Large-for-gestational-age newborns C) Appropriate-for-gestational-age newborns D) Low-birthweight newborns

C Appropriate-for-gestational-age newborns are at the lowest risk for any problems. The other categories all have an increased risk of complications.

The nurse encourages the mother of a healthy newborn to put the newborn to the breast immediately after birth for which reason? A) To aid in maturing the newborn's sucking reflex B) To encourage the development of maternal antibodies C) To facilitate maternal-infant bonding D) To enhance the clearing of the newborn's respiratory passages

C Breast-feeding can be initiated immediately after birth. This immediate mother-newborn contact takes advantage of the newborn's natural alertness and fosters bonding. This contact also reduces maternal bleeding and stabilizes the newborn's temperature, blood glucose level, and respiratory rate. It is not associated with maturing the sucking reflex, encouraging the development of maternal antibodies, or aiding in clearing of the newborn's respiratory passages.

The nurse assesses a 1-day-old newborn. Which finding indicates that the newborn's oxygen needs aren't being met? A) Respiratory rate of 54 breaths/minute B) Abdominal breathing C) Nasal flaring D) Acrocyanosis

C Nasal flaring is a sign of respiratory difficulty in the newborn. A rate of 54 breaths/minute, diaphragmatic/abdominal breathing, and acrocyanosis are normal findings.

The parents of a preterm newborn being cared for in the neonatal intensive care unit (NICU) are coming to visit for the first time. The newborn is receiving mechanical ventilation and intravenous fluids and medications and is being monitored electronically by various devices. Which action by the nurse would be most appropriate? A) Suggest that the parents stay for just a few minutes to reduce their anxiety. B) Reassure them that their newborn is progressing well. C) Encourage the parents to touch their preterm newborn. D) Discuss the care they will be giving the newborn upon discharge.

C The NICU environment can be overwhelming. Therefore, the nurse should address their reactions and explain all the equipment being used. On entering the NICU, the nurse should encourage the parents to touch, interact, and hold their newborn. Doing so helps to acquaint the parents with their newborn, promotes self-confidence, and fosters parent-newborn attachment. The parents should be allowed to stay for as long as they feel comfortable. Reassurance, although helpful, may be false reassurance at this time. Discussing discharge care can be done later once the newborn's status improves and plans for discharge are initiated.

The nurse is teaching a pregnant woman with type 1 diabetes about her diet during pregnancy. Which client statement indicates that the nurse's teaching was successful? A) "I'll basically follow the same diet that I was following before I became pregnant." B) "Because I need extra protein, I'll have to increase my intake of milk and meat." C) "Pregnancy affects insulin production, so I'll need to make adjustments in my diet." D) "I'll adjust my diet and insulin based on the results of my urine tests for glucose."

C) "Pregnancy affects insulin production, so I'll need to make adjustments in my diet." In pregnancy, placental hormones cause insulin resistance at a level that tends to parallel growth of the fetoplacental unit. Nutritional management focuses on maintaining balanced glucose levels. Thus, the woman will probably need to make adjustments in her diet. Protein needs increase during pregnancy, but this is unrelated to diabetes. Blood glucose monitoring results typically guide therapy.

A client at 28 weeks gestation is admitted to the labor and birth unit. Which test would most likely be used to assess the client's comprehensive fetal status? a) Ultrasound for physical structure b) Nonstress test (NST) c) Biophysical profile (BPP) d) Amniocentesis

C. Biophysical profile is a comprehensive test that would be used to assess the client's fetal status at 28 weeks gestation. Ultrasound for physical structure is limited to identifying the growth and development of the fetus, and does not assess for other parameters of fetal well-being. Women with a high-risk factor will probably begin having NSTs at 30-32 weeks gestation and at frequent intervals for the remainder of the pregnancy. Amniocentesis late in pregnancy is used to test for lung maturity, not overall fetal status in labor, and when performed earlier it is used to test for specific disorders.

A pregnant client has a history of asthma. After reveiwing the possible medications that may be prescribed during her pregnancy to control her asthma, the nurse determines additional teaching is needed when the client identifies which drug as being used? A. albuterol B. salmeterol C. oral prednisone D. budesonide

C. oral prednisone

A woman gave birth to a newborn via vaginal birth with the use of a vacuum extractor. The nurse would be alert for which possible effect in the newborn? A. asphyxia B. clavicular fracture C. cephalhematoma D. central nervous system injury

C. cephalhematoma explanation: Use of forceps or a vacuum extractor poses the risk of tissue trauma, such as ecchymoses, facial and scalp lacerations, facial nerve injury, cephalhematoma, and caput succedaneum. Asphyxia may be related to numerous causes, but it is not associated with use of a vacuum extractor. Clavicular fracture is associated with shoulder dystocia. Central nervous system injury is not associated with the use of a vacuum extractor.

A client is diagnosed with gestational hypertension and is receiving magnesium sulfate. Which finding would the nurse interpret as indicating a therapeutic level of medication? A. urinary output of 20 mL per hour B. respiratory rate of 10 breaths/minute C. deep tendons reflexes 2+ D. difficulty in arousing

C. deep tendons reflexes 2+ explanation: With magnesium sulfate, deep tendon reflexes of 2+ would be considered normal and therefore a therapeutic level of the drug. Urinary output of less than 30 mL, a respiratory rate of less than 12 breaths/minute, and a diminished level of consciousness would indicate magnesium toxicity.

Which information on a client's health history would the nurse identify as contributing to the client's risk for an ectopic pregnancy? A. use of oral contraceptives for 5 years B. ovarian cyst 2 years ago C. recurrent pelvic infections D. heavy, irregular menses

C. recurrent pelvic infections explanation: In the general population, most cases of ectopic pregnancy are the result of tubal scarring secondary to pelvic inflammatory disease. Oral contraceptives, ovarian cysts, and heavy, irregular menses are not considered risk factors for ectopic pregnancy.

The nurse is preparing a prenatal client for a transvaginal ultrasound. What nursing action should the nurse include in the preparations? Select all that apply. a)Advise the client to empty her bladder. b)Encourage the client to drink 1.5 quarts of fluid. c)Apply transmission gel over the client's abdomen. d)Place client in lithotomy position.

D After having the client void, assist her to a lithotomy position for a transvaginal ultrasound. Preparation for a transabdominal ultrasound includes encouraging the client to drink 1.5 quarts of fluid, maintaining a full bladder, and applying transmission gel over the client's abdomen.

What is the best intervention a nurse can utilize to promote parent-infant attachment? a)Allow for privacy. b)Contact support families that have been through the same diagnosis with their own child and allow time to discuss the situation. c)Provide an extensive handbook with information related to the preterm newborn. d)Encourage rooming in.

D All will help strengthen the attachment bond, but the best answer would be to encourage rooming in. Rooming in can provide a great opportunity for the stable preterm infant and family to get acquainted; it offers both privacy and readily available help.

The nurse is caring for a pregnant client who indicates that she is fond of meat, works with children, and has a pet cat. Which of the following instructions should the nurse give this client to prevent toxoplasmosis? (Select all that apply). a. Eat meat cooked to 160 degrees b. Avoid cleaning the cat's litter box c. Keep the cat outdoors at all times d. Avoid contact with children when they have a cold e. Avoid outdoor activities such as gardening

a. Eat meat cooked to 160 degrees b. Avoid cleaning the cat's litter box e. Avoid outdoor activities such as gardening

The client in labor experiences a spontaneous rupture of membranes. What information related to this event must the nurse include in the client's record? a. Fetal heart rate b. Pain level c. Test results ensuring that the fluid is not urine d. The client's understanding of the event

a. Fetal heart rate

Which is an essential part of nursing care for a laboring client? a. Helping the woman manage the pain b. Eliminating the pain associated with labor c. Feeling comfortable with the predictable nature of intrapartal care d. Sharing personal experiences regarding labor and birth to decrease her anxiety

a. Helping the woman manage the pain

Which comfort measure should a nurse use to assist a laboring woman to relax? a. Recommend frequent position changes. b. Palpate her filling bladder every 15 minutes. c. Offer warm wet cloths to use on the client's face and neck. d. Keep the room lights lit so the client and her coach can see everything.

a. Recommend frequent position changes.

The nurse is caring for a client in the fourth stage of labor. Which assessment findings should the nurse identify as a potential complication? (Select all that apply.) a. Soft boggy uterus b. Maternal temperature of 99° F c. High uterine fundus displaced to the right d. Intense vaginal pain unrelieved by analgesics e. Half of a lochia pad saturated in the first hour after birth

a. Soft boggy uterus c. High uterine fundus displaced to the right d. Intense vaginal pain unrelieved by analgesics

When using the second Leopold's maneuver in fetal assessment, the nurse would palpate (the): a. both sides of the maternal abdomen. b. lower abdomen above the symphysis pubis. c. both upper quadrants of the maternal abdomen . d. lower abdomen for flexion of the presenting part.

a. both sides of the maternal abdomen.

If a woman's fundus is soft 30 minutes after birth, the nurse's first response should be to: a. massage the fundus. b. take the blood pressure. c. notify the physician or nurse-midwife. d. place the woman in Trendelenburg position.

a. massage the fundus.

Which interventions should be performed in the birth room to facilitate thermoregulation of the newborn? (Select all that apply.) a. Place the infant covered with blankets in the radiant warmer. b. Dry the infant off with sterile towels. c. Place stockinette cap on infant's head. d. Bathe the newborn within 30 minutes of birth. e. Remove wet linen as needed.

b. Dry the infant off with sterile towels. c. Place stockinette cap on infant's head. e. Remove wet linen as needed.

A gravida 1, para 0, 38 weeks' gestation is in the transition phase of labor with SROM and is very anxious. Vaginal exam, 8 cm, 100% effaced, -1 station vertex presentation. She wants the nurse to keep checking her by performing repeated vaginal exams because she is sure that she is progressing rapidly. What is the best response that the nurse can provide to this client at this time? a. Performing more frequent vaginal exams will not make the labor go any quicker. b. Even though she is in transition, frequent vaginal exams must be limited because of the potential for infection. c. Tell the client that she will check every 30 minutes. d. Medicate the client as needed for anxiety so that the labor can progress.

b. Even though she is in transition, frequent vaginal exams must be limited because of the potential for infection.

The nurse is caring for a low-risk client in the active phase of labor. At which interval should the nurse assess the fetal heart rate? a. Every 15 minutes b. Every 30 minutes c. Every 45 minutes d. Every 1 hour

b. Every 30 minutes

Which nursing assessment indicates that a woman who is in the second stage of labor is almost ready to give birth? a. Bloody mucous discharge increases. b. The vulva bulges and encircles the fetal head. c. The membranes rupture during a contraction. d. The fetal head is felt at 0 station during the vaginal examination.

b. The vulva bulges and encircles the fetal head.

A nursing priority during admission of a laboring client who has not had prenatal care is: a. obtaining admission labs. b. identifying labor risk factors. c. discussing her birth plan choices. d. explaining importance of prenatal care.

b. identifying labor risk factors.

At 1 minute after birth, the nurse assesses the newborn to assign an Apgar score. The apical heart rate is 110 bpm, and the infant is crying vigorously with the limbs flexed. The infant's trunk is pink, but the hands and feet are blue. The Apgar score for this infant is: a. 7. b. 8. c. 9. d. 10.

c. 9.

The health care provider has asked the nurse to prepare for an amniotomy. What is the nurse's priority action with this procedure? a. Perform Leopold's maneuvers. b. Determine the color of the amniotic fluid. c. Assess the fetal heart rate immediately after the procedure. d. Prepare the patient for a change in her pain level after the procedure.

c. Assess the fetal heart rate immediately after the procedure.

A woman arrives to the labor and birth unit at term. She is greeted by a staff nurse and a nursing student. The student reviews the initial intake assessment with the staff nurse. Which action will the staff nurse have to correct? a. Obtain a fetal heart rate. b. Determine the estimated due date. c. Auscultate anterior and posterior breath sounds. d. Ask the client when she last had something to eat.

c. Auscultate anterior and posterior breath sounds.

The nurse notes that a client who has given birth 1 hour ago is touching her infant with her fingertips and talking to him softly in high-pitched tones. Based on this observation, which action should the nurse take? a. Request a social service consult for psychosocial support. b. Observe for other signs that the mother may not be accepting of the infant. c. Document this evidence of normal early maternal-infant attachment behavior. d. Determine whether the mother is too fatigued to interact normally with her infant.

c. Document this evidence of normal early maternal-infant attachment behavior.

Which nursing diagnosis would take priority in the care of a primipara client with no visible support person in attendance who has entered the second stage of labor after a first stage of labor lasting 4 hours? a. Fluid volume deficit (FVD) related to fluid loss during labor and birth process b. Fatigue related to length of labor requiring increased energy expenditure c. Acute pain related to increased intensity of contractions d. Anxiety related to imminent birth process

d. Anxiety related to imminent birth process

The nurse examines a primipara's cervix at 8-9/100%/+2; it is tight against the fetal head. The patient reports a strong urge to bear down. What is the nurse's priority action? a. Palpate her bladder for fullness. b. Assess the frequency and duration of her contractions. c. Determine who will stay with the patient for the birth. d. Encourage the patient to exhale in short breaths with contractions.

d. Encourage the patient to exhale in short breaths with contractions.

Which client at term should go to the hospital or birth center the soonest after labor begins? a. Gravida 2, para 1, who lives 10 minutes away b. Gravida 1, para 0, who lives 40 minutes away c. Gravida 2, para 1, whose first labor lasted 16 hours d. Gravida 3, para 2, whose longest previous labor was 4 hours

d. Gravida 3, para 2, whose longest previous labor was 4 hours

Which should the nurse recognize as being associated with fetal compromise? a. Active fetal movements b. Fetal heart rate in the 140s c. Contractions lasting 90 seconds d. Meconium-stained amniotic fluid

d. Meconium-stained amniotic fluid

The gynecologist performs an amniotomy. What will the nurse's role include immediately following the procedure? a. Assessing for ballottement b. Conducting a pH and/or fern test c. Labeling of specimens for chromosomal analysis d. Recording the character and amount of amniotic fluid

d. Recording the character and amount of amniotic fluid

A 25-year-old primigravida client is in the first stage of labor. She and her husband have been holding hands and breathing together through each contraction. Suddenly, the client pushes her husband's hand away and shouts, "Don't touch me!" This behavior is most likely: a. abnormal labor. b. a sign that she needs analgesia. c. normal and related to hyperventilation. d. common during the transition phase of labor.

d. common during the transition phase of labor.

A primigravida at 39 weeks of gestation is observed for 2 hours in the intrapartum unit. The fetal heart rate has been normal. Contractions are 5 to 9 minutes apart, 20 to 30 seconds in duration, and of mild intensity. Cervical dilation is 1 to 2 cm and uneffaced (unchanged from admission). Membranes are intact. The nurse should expect the client to be: a. discharged home with a sedative. b. admitted for extended observation. c. admitted and prepared for a cesarean birth. d. discharged home to await the onset of true labor.

d. discharged home to await the onset of true labor.

After teaching a pregnant woman with iron deficiency anemia about nutrition, the nurse determines that the teaching was successful when the woman identifies which foods as being good sources of iron in her diet? Select all that apply. dried fruits peanut butter meats milk white bread

dried fruits peanut butter meats explanation: Foods high in iron include meats, green leafy vegetables, legumes, dried fruits, whole grains, peanut butter, bean dip, whole-wheat fortified breads, and cereals.

Assessment of a postpartum client reveals a firm uterus with bright-red bleeding and a localized bluish bulging area just under the skin at the perineum. The woman also reports significant pelvic pain and is experiencing problems with voiding. The nurse suspects which condition? hematoma laceration bladder distention uterine atony

hematoma explanation: The woman most likely has a hematoma based on the findings: firm uterus with bright-red bleeding; localized bluish bulging area just under the skin surface in the perineal area; severe perineal or pelvic pain; and difficulty voiding. A laceration would involve a firm uterus with a steady stream or trickle of unclotted bright-red blood in the perineum. Bladder distention would be palpable along with a soft, boggy uterus that deviates from the midline. Uterine atony would be noted by a uncontracted uterus.

A client experienced prolonged labor with prolonged premature rupture of membranes. The nurse would be alert for which condition in the mother and the newborn? infection hemorrhage trauma hypovolemia

infection explanation: Although hemorrhage, trauma, and hypovolemia may be problems, the prolonged labor with the prolonged premature rupture of membranes places the client at high risk for a postpartum infection. The rupture of membranes removes the barrier of amniotic fluid, so bacteria can ascend.


Conjuntos de estudio relacionados

The New Deal: First Hundred Days, Alphabet Soup Programs (New Deal and Second New Deal)

View Set

ECO285 Final Exam Practice Questions

View Set

MUSIC 118C - Final Lessons 21-39

View Set

Chapter 5 Methods of Transferring Property

View Set